Vous êtes sur la page 1sur 163

Short Notes

and Short Cases


in Gynaecology
Short Notes
and Short Cases
in Gynaecology

UN Panda
MD
Senior Physician
New Delhi

JAYPEE BROTHERS
Medical Publishers (P) Ltd
New Delhi
Published by
Jitendar P. Vij
Jaypee Brothers Medical Publishers (P) Ltd
EMCA House, 23/23B Ansari Road, Daryaganj
New Delhi 110 002, India
Phones: 23272143, 23272703, 23282021, 23245672, 23245683
Fax: 011-23276490
e-mail: jpmedpub@del2.vsnl.net.in
Visit our web site: http://www.jpbros.20m.com
Branches
202 Batavia Chambers, 8 Kumara Kruppa Road, Kumara
Park East, Bangalore 560 001, Phones: 2285971, 2382956
Tele Fax: 2281761, e-mail: jaypeebc@bgl.vsnl.net.in
282 IIIrd Floor, Khaleel Shirazi Estate, Fountain Plaza
Pantheon Road, Chennai 600 008, Phone: 28262665
Fax: 28262331, e-mail: jpmedpub@md3.vsnl.net.in
4-2-1067/1-3, Ist Floor, Balaji Building, Street No.6, Ramkote
Cross Road, Hyderabad 500 095, Phone: 55610020, 24758498,
Fax: 24758499, e-mail: jpmedpub@rediffmail.com
1A Indian Mirror Street, Wellington Square
Kolkata 700 013, Phone: 2451926 Fax: 2456075
e-mail: jpbcal@cal.vsnl.net.in
106 Amit Industrial Estate, 61 Dr SS Rao Road, Near MGM
Hospital Parel, Mumbai 400 012, Phones: 24124863,
24104532, Fax: 24160828
e-mail: jpmedpub@bom7.vsnl.net.in
Short Notes and Short Cases in Gynaecology
2003, UN Panda
All rights reserved. No part of this publication should be reproduced,
stored in a retrieval system, or transmitted in any form or by any means:
electronic, mechanical, photocopying, recording, or otherwise, without
the prior written permission of the author and the publisher.
This book has been published in good faith that the material provided
by author is original. Every effort is made to ensure accuracy of
material, but the publisher, printer and author will not be held
responsible for any inadvertent error(s). In case of any dispute, all
legal matters to be settled under Delhi jurisdiction only.

First Edition : 2003


Publishing Director: RK Yadav
ISBN 81-8061-133-7
Typeset at JPBMP typesetting unit
Printed at Lordson Publishers (P) Ltd., C-5/19, RP Bagh,
Delhi 110 007
Preface

Short notes and short cases in gynaecology is a hand-


book especially structured for undergraduate medical
students. The subject matter has been presented in ques-
tion - answer format for quick revision and easy
reproduction. Students will find the book handy and very
concise, to be quick revised before examination. The short
cases discussed in details are the ones commonly pre-
sented to undergraduate students at examination. So the
students will have the feel of clinical and viva voce, while
going through them.
I am sure the book will be of immense help to under-
graduates. All suggestions for improvement are cordially
welcome.

UN Panda
Contents

SHORT NOTES

1. Vulvovaginitis ... 3
2. Disorders of Menstruation ... 10
3. Pelvic Inflammatory Disease ... 15
4. Abnormal Uterine Bleeding ... 18
5. Pap Smear ... 21
6. Uterine Leiomyomas ... 25
7. Pelvic Relaxation ... 29
8. Uterine Displacement ... 32
9. Urinary Problems ... 35
10. Adnexal Mass ... 38
11. Ectopic Pregnancy ... 42
12. Endometriosis ... 46
13. Acute and Chronic Pelvic Pain ... 51
14. Contraception ... 54
15. Abortion ... 57
16. Vulva, Vagina, Cervix ... 67
17. Carcinoma Vulva ... 71
18. Carcinoma Cervix ... 74
19. Carcinoma Endometrium ... 80
20. Gestational Trophoblastic Disease ... 82
21. Tumors of Ovary ... 85
22. Infertility ... 93
23. Anovulation, Polycystic Ovary ... 95
24. IVF-ET ... 97
25. Menopause ... 100
viii Short Notes and Short Cases in Gynaecology

SHORT CASES
Case 1 ... 105
Case 2 ... 112
Case 3 ... 118
Case 4 ... 123
Case 5 ... 128
Case 6 ... 132
Case 7 ... 136
Case 8 ... 140
Case 9 ... 147
Case 10 ... 151
Short Notes
1 Vulvovaginitis

1. Normal vaginal discharge


A physiologic vaginal discharge consists of cervical and vagi-
nal secre-tions, epithelial cells and bacterial flora. It is white,
odorless and does not cause burning, itching or discomfort.
It increases in quantity prior to menstruation. The pH is
3.8-4.2.
Composition of normal vaginal flora
Lactobacillus species (Doderleins bacillus), an aerobic gram
positive rod is the most common content of vaginal flora.
Others are Gardenerella vaginalis, anaerobes, diphtheroids,
strepto and staphylococci, candida and mycoplasma species.
2. Types of vulvitis
Inflammatory disease mainly affecting vulva itself.
Pyogenic infection of the hair follicles.
Infected sebaceous cysts in vulva
Infection due to traumamay be coital
accidental, obstetric operation or by scratching
Venereal infection of vulva
Gonococcal vulvovaginitis
Senile vulvovaginitis.
Inflammatory lesion primarily affecting structures in
vulva. eg. Bartholinitis
Secondary inflammatory disease of vulva.
Trichomonas, Moniliasis, Diabetic vulvitis, Vulvitis
secondary to cervical lesion, Vulvitis secondary to
urinary to rectal fistula.
Skin conditions affecting vulva.
Tinea cruris, Scabies, Pediculosis, Herpes zoster,
4 Short Notes and Short Cases in Gynaecology

Contact dermatitis, Neurodermatitis, Threadworm


infestation, Seborrhoeic dermatitis, Psoriasis.
3. Causes of pruritus vulvae
Trichomoniasis or moniliasis
Skin conditions affecting vulva
Constitutional toxic states like jaundice and uremia
Deficiency diseasesIron deficiency anemia, achlo-
rhydria, Vit-A and B deficiency
Diabetes
Nonspecific vaginitis and cervicitis
Carcinoma vulva
Senile vaginitis
PsychogenicIdiopathic.
4. Treatment of pruritus vulvae
Strict hygiene of vulva including cutting short of hair
or shaving
Good sleep
Avoidance of irritating antiseptic lotion and cheap
soaps
Application of lotion like lotio plumbi or ointment of
3 percent carbolic and 5 percent menthol
Good result is obtained by to 1 percent hydrocorti-
sone oint. When there is no acute infection
Administration of Estrogen in case of menopausal
women.
5. Kraurosis vulvae
This is a condition of primary atrophy of vulva. It is a pro-
gressive atrophy of the skin of the vulva resulting in gradual
stenosis of vaginal introitus. It is premalignant.
Symptoms
Local discomfort
Difficulty during coitus
Sometimes urinary symptoms.
Treatment
Application of zinc oxide ointment
Estrogen cream or oral oestrogens.
Vulvovaginitis 5

6. Tumors associated with vulva


Papilloma of vulva
Fibroma
Fibromyoma
Lipoma
Sarcoma of the vulva
Malignant melanoma
Carcinoma of vulva
Endometrioma.
7. Predisposition to cervicitis
Multi-parity
Frequent abortion
Foreign body or cervical trauma by instruments
Gonorrhea
Poor vaginal hygiene
Hypo-estorgenism
Vitamin deficiency.
8. Benign tumors of vulva
Nevi : They are pigmented lesions composed
of clusters of nevus cells. These lesions
should be treated with wide local exci-
sion if undergoing color change.
Leiomyomas : Leiomyomas may arise from smooth
muscle in erectile vulval structures or
remnants of round ligament.
Fibromas and : Fibromas are similar to leiomyomas but
lipomas of mesordermal origin. Fibromas and
lipomas need wide excision.
Glomus tumor : They occur in dermis and are painful.
Hidroadenomas : They are benign sweat-gland tumors,
small firm with pointed center. Simple
excision is curative.
9. Vulval lesions in sexually transmitted diseases
Condyloma : Caused by human papilloma virus, vary
acuminata from single to numerous lesions involv-
ing vulva, perineum, vagina and cervix.
Podophyllum application, cautery, laser
and cryosurgery can cure them.
6 Short Notes and Short Cases in Gynaecology

Syphilis of vulva : Syphilitic chancre commonly occurs on


vagina and cervix but occasionally on
vulva. They are painless linear firm
ulcers with raised edges with crater-like
appearance. Secondary lesions appear
as moist greyish patches on perineum
called condylomalata.
Granuloma : Caused by a Donovan bacillus. It starts
inguinale as a small papule on vulva and spreads
seripigenously till entire vulvoperineal
area is a mass of chronic granuloma-
tous tissue. Symptoms are itching, burn-
ing pain and discharge. Tetracycline is
very effective.
Lympho : It is caused by LGV virus where the pri
granuloma mary vulval lesion escapes attention but
venereum to be followed by marked inguinal
adenitis and often pleuropericarditis,
meningitis and arthritis. Rectal and
vaginal stricture may occur later. Tetra-
cycline is effective.
Chancroid : It is caused by Ducreys bacillus and is
very painful. Inguinal adenitis follows.
Tetracycline and broad spectrum anti-
biotic are curative.
10. Cystic lesions of vulva and vagina
Vulva Vagina
Sebaceous cyst Inclusion cyst
Inclusion cyst Gartner duct cyst
Bartholins gland cyst Endometriosis
11. Vestibular adenitis
Though it is a debatable lesion, it causes considerable perineal
pain and dyspareunia in otherwise healthy women. Local hy-
drocortisone or lignocaine ointment may bring relief else lo-
cal excision be considered but in many the syndrome recurs.
12. Lichen sclerosus et atrophicus
A chronic localised lesion of vulva and perineum of patients
beyond 50 with remissions and exacerbations, it appears as
Vulvovaginitis 7

an ivory white smooth surface causing itching and burning.


Biopsy is confirmatory showing dens subepidermal collagen
layer and dense parakeratosis. Neck, axillae, and shoulders
can have similar lesions.
13. Urethral lesions presenting as vaginal masses
Urethral diverticulum : It bulges into vagina as a smooth
cystic pouch to cause dribbling
after urination. Infection can cause
pain and dysuria.
Suburethral abscess : It can be due to infection of ure-
thral diverticulum or periurethral
glands. Drainage to vaginal side is
required.
Urethral caruncle : It can be haemangiomatous or
chronic granulomatous appearing
as a small red protruding lesion from
edge of urethral orifice, mimicking
cancer or prolapsed urethral mu-
cosa. Removal by cautery/laser is
required if symptomatic.
14. Vulvar hematoma
Vulvar hematoma dissects widely beneath the fascia of
perineum hence drainage with packing is required to achieve
hemostasis since bleeding vessel is often obscure.
15. Bartholin cyst/abscess
Bartholin glands are paired mucus secreting vulval glands
draining to posterior introitus. Obstruction of duct near open-
ing causes cyst formation in vulva. Most cysts are unilocular
and unilateral. Infection with gonococci, E. coli, proteus and
anaerobes produces abscess. Symptomatic recurrent cysts
need marsupialization or laser vaporization of cyst wall.
Bartholins abscess needs excision and drainage. A word
catheter can be placed to maintain patency.
Bartholins gland excision is generally not recommended
because it causes lot of bleeding, haematoma, cellulitis,
incomplete removal and hence recurrence. In postmenopausal
women Bartholins gland cyst may represent a malignant
tumor.
8 Short Notes and Short Cases in Gynaecology

16. Word catheter


It is a single lumen catheter with inflatable balloon tip. A small
incision is made at the introitus and the balloon tip is intro-
duced to Bartholin cyst and is insuflated with 2 to 4 ml of water.
The catheter is left in place for 3 to 6 weeks, then deflated and
removed. A permanent ostium remains to drain the gland.
17. Vaginitis
The three most common causes of vaginitis are: (i) candida
(yeast, monilia), (ii) Trichomonas vaginalis and (iii) syner-
gistic bacterial infections (bacterial vaginosis). Specific etio-
logic agent can be suspected from presenting symptoms and
sign microscopic examination of discharge and vaginal pH.
18. Yeast vaginitis
Pruritus, dysuria, cottage cheese like discharge are charac-
teristic. KOH preparation shows the characteristic pseudo-
hyphae. Diabetes and immunosuppression (HIV) predispose
to yeast infection. Fluconazole 200 mg orally for 5 days or
intravaginal application of miconazole, clotrimazole,
butoconazole, tercanozole for 3-7 days are curative.
19. Trichomonal vaginitis
The characteristic symptom is profuse, malodorous and
greenish-gray frothy vaginal discharge. Saline smear shows
the unicellular flagellated organism. Metronidazole 500 mg
twice daily for 7 days is curative. Husband needs to be treated
with similar dose. Metronidazole 2 gm single dose or tinidazole
2 gm/secnidazole 2 gm are the alternatives.
20. Bacterial vaginosis
The patient complains of increased gray-coloured vaginal
discharge with fishy odor. Microscopic examination of saline
smear shows the characteristic clue cells, the vaginal epi-
thelial cells studded with cocobacilli, the Gardenerella
vaginalis obscuring the cell border. Treatment is with met-
ronidazole 500 mg bid for 7 days or clindamycin 300 mg bid
for 7 days. Husband should be treated. Bacterial vaginosis
can cause preterm labor or PROM.
21. Chlamydial genital infection
Chlamydia trachomatis causes mucopurulent cervicitis with
increased vaginal discharge. It should be suspected when
Vulvovaginitis 9

microscopic examination of saline smear reveals numerous


white blood cells but no clue cells, yeast or trichomonads.
Doxycycline 100 mg bd for 7 days, azithromycin 1 gm single
dose, ofloxacin 300 mg bd for 7 days are curative. Husband
also to be treated.
22. Cocktail therapy of vaginitis:
23. Atrophic vaginitis
Estrogen deficiency after menopause or oophorectomy
causes atrophy and subsequent inflammation of vaginal
mucosa. Patient has dryness, itching and dyspareunia and
occasionally vaginal bleeding. Vaginal mucosa is thin with
loss of rugations. Oral estrogen replacement or local estro-
gen cream are of help.
24. Genital herpes
Ninety percent genital herpes is by HSV II and 10 percent is
by HSV I; the reverse is true for orofacial herpes. Initial infec-
tion is mainly asymptomatic, the virus is causing prolonged
latent infection of the inervating root ganglia cells (S3S4).
Sometimes primary infection causes widespread painful
vesicles with fever and aseptic meningitis, taking 12-21 days
to heal. Recurrent herpes is always localized and heals in 2-4
days. HSV II causes more severe and recurrent infection than
HSV I. Classic herpetic lesions are dew drops on rose petal
appearance but in genital herpes painful ulcerations, follicu-
litis type lesions are more common. Herpetic cervicitis causes
white-gray area of necrosis mimicking cancer. HSV II and
HPV are known to cause genital cancer in long-run.
Primary infection or recurrent sever genital infection needs
treatment with acyclovir, valcyclovir or famcyclovir (the later
two have better pharmacokinetic with twice dosing). The
viral thymidine kinase activates these drugs to prevent viral
replication. Oral dose of acyclovir is 200 mg 5 times daily. If
hepatitis or encephalitis is associated 5 mg/kg IV tid is
recommended. Patients with frequent recurrence can be given
suppressive therapy with 800 mg daily for 1 year. Serum crea-
tinine be estimated with prolonged acyclovir therapy. When
STD are associated HIV transmission after intercourse is
increased.
2 Disorders
of Menstruation

1. Dysmenorrhoea and types


Painful menstruation (dysmenorrhoea) can be primary (in
absence of pelvic disease) or secondary (with pelvic patho-
logy).
Primary Secondary
Essential Acquired Mechanical Membranous
Psychogenic Congestive Tubal Obstructive
Spasmodic Inflammatory Uterine Ovarian
2. Primary dysmenorrhoea
The pain is cramping located in lower abdomen, often radiating
to upper thigh and back. It starts 6-12 months after menarche.
The pain starts several hours before or with menstrual flow
and abates within 48-72 hours. Associated symptoms are
nausea and vomiting, headache, fatigue, low backache and
diarrhoea. Though cause of this pain is uncertain, uterine
ischaemia and prostaglandins are blamed. Oral contraceptive
pills reduce endometrial thickness and revert endometrium to
proliferative stage, thus reducing prostaglandin level and pain.
Prostaglandin inhibitors are also equally effective and be pre-
scribed to young or those wanting pregnancy.
a. Functional i. Unusual contraction of cervix and
relaxation of uterus
ii. Hormonal imbalance of oestrogen
iii. Lack of thrombolysin enzyme
iv. Psychological fear
v. Ischaemic factor
b. Structural i. Hypoplastic uterus or infantile uterus
ii. Mal-developed or Bi-cornuate uterus.
Disorders of Menstruation 11

3. Symptoms of spasmodic dysmenorrhoea


Pain over lower abdomen which starts few hours
before onset of period and lasts for about 12 hrs.
Increase in intensity of pain.
Sometimes faintness, collapse, nausea, vomiting
Some degree of menstrual irregularity in duration and
cycle.
Sterile patients do not suffer from this type of dys-
menorrhoea.
4. Treatment of spasmodic dysmenorrhoea
Symptomatic pain killers and anti-spasmodics.
Physical exercisegames, gymnastics
Change of climateopen air
Constipation if persists is treated
Marriage and child birth relieve spasmodic dysmen-
orrhoea.
Alcohol in western countries is a common remedy
Use of estrogen preparation like stilbesterol1 mg
thrice daily before 15 days of onset of period
In older patient with failure in conservative treatment
operative measures like
i. D and C
ii. Presacral sympathectomy
iii. Hysterectomy as a terminal procedure.
5. Congestive dysmenorrhoea
This is a form of painful menstruation occurring due
to congestion in pelvis. The pain starts 3-5 days be-
fore onset of mense and is relieved by menstrual flow.
6. Causes of congestive dysmenorrhoea
Constipation
Tumours in pelvis
Endometriosis
Inflammatory diseases such as salpingitis, oophritis
parametritis, pelvic adhesion
Acquired retroversion of uterus.
7. Treatment of congestive dysmenorrhoea
Diet and avoidance of excess carbohydrate
Avoidance of strong purgatives
12 Short Notes and Short Cases in Gynaecology

Physical exercises
Anti-spasmodic drugs
8. Membranous dysmenorrhoea
This is an extreme type of dysmenorrhoea in which the whole
of endometrium is shed-out in the form of a membrane. It is
rare and runs in families and after pregnancy.
9. Ovarian dysmenorrhoea
This is a form of dysmenorrhoea in which pain is felt from
umbilical region to the inguinal canal due to some disease of
ovary. Ovarian sympathectomy and pre-sacral neurectomy is
the line of treatment.
10. Diseases to be considered when evaluating secondary
dysmenorrhoea:
1. Endometriosis/endometritis
2. Ruptured ovarian cyst
3. Chronic pelvic pain
4. Torsion of ovary, ovarian cyst
5. Adenomyosis, leiomyoma
6. Ruptured ectopic pregnancy
7. Pelvic inflammatory disease
8. Appendicitis/diverticulitis.
Laparoscopy, hysteroscopy pelvic ultrasound and
hysterosalpingogram should be done to exclude only pelvic
pathology as secondary cause of dysmenorrhoea.
11. Amenorrhoeaprimary and secondary
Primary amenorrhoea: Absence of menses by 16 years of
age.
Secondary dysmenorrhoea: Absence of menses longer
than 6 to 12 months or duration of three previous cycle inter-
vals.
12. Causes of amenorrhoea
Physiological
Prepuberty
Pregnancy
Lactation
Menopause
Disorders of Menstruation 13

Pathological
Primary
Imperforate hymen
Non-canalization of lower genital tract like cervix or vagina
Absence or gross hypoplasia of uterus
Turners syndrome
Secondary
Acquired obstruction of genital tract
Hysterectomy
Disorder of ovary
Pituitary disorders
Disorders of adrenal glands
Hyperthyroidism
Diabetes mellitus
Miscellaneous disease like Carcinoma, advanced tuber-
culosis, severe anemia, and malnutrition.
Exercise, stress, athletics
Anorexia nervosa
Drugs-TCA, antipsychotics, benzodiazepines, opiates,
barbiturates
13. Pituitary causes of amenorrhoea
Sheehans syndrome and Simmonds syndrome
Prolactinomasaccount for 10 to 20 percent cases of
amenorrhoea.
Acromegaly and Cushings disease
14. Work of secondary amenorrhoea
Serum prolactin, T3 T4 TSH
CT scan of pituitary fossa
US of pelvis
Dilatation and curettage with biopsy.
15. Premature ovarian failure
Amenorrhoea secondary to ovarian failure before 40 years is
premature ovarian failure. Many patients have autoimmune
diseases like Hashimotos, Addisons, hypoparathyroidism.
If patient is less than 25 years, karyotyping for 46 XX/46 XY
14 Short Notes and Short Cases in Gynaecology

mosaic. If so gonads must be removed. Antithyroid anti-


bodies, antinuclear antibody and 24 hr cortisol levels should
be done.
16. Enzyme deficiency causing amenorrhoea
17 alpha hydroxylase
17 to 20 desmolase.
17. Progesterone challenge test
100 to 200 mg of progesterone IM or 10 mg medroxypro-
gesterone daily for 5 days is given. Bleeding within 2 weeks
indicates that serum estradiol level > 40 pg/ml, normal FSH,
LH and endometrial function. If prolactin level and T3T4 TSH
are normal, then patient is anovulatory.
18. Approach to Patient with Negative Progesterone Chal-
lenge
A coarse of conjugated estrogen 2.5 mg for 21 days, then
progesterone challenge. If no bleed think of endometrial
fibrosis or Asherman syndrome. Raised FSH indicates primary
ovarian failure, if normal, CT be done to exclude craniopharyn-
gioma and granulomatous disease (tuberculosis, sarcoid) of
pituitary.
3 Pelvic Inflammatory
Disease

1. PID and its causes


PID is an infection of upper genital tract in women. The infec-
tion starts in the lower genital tract and ascends up. It usually
involves sexually transmitted organisms and often other
organisms endogenous to lower genital tract. Parametrium is
often involved and infection may migrate along the perito-
neum to upper abdomen to cause adhesion (violin strings) of
liver capsule to diaphragm (Fitz-High Curtis syndrome).
The primary organisms are gonococci and chlamydia.
Chlamydia infection is often silent but gonococcal infection
is abrupt onset with much local and constitutional symp-
toms. Endometrial biopsy, IUD insertion, hysteroscops and
such procedure can lead to PID. However, pre and postmeno-
pausal women and pregnancy provide protection.
Aerobic Anaerobes
Chlamydia trachomatis Bacteroids
N. gonorrhoeae Peptococcus
Gardenerella vaginalis
Peptostreptococcus
Streptococcus
H. influenzae
Proteus mirabilis
Klebsiella species
2. Diagnosis of PID
The triad of lower abdominal pain and tenderness, cervical
motion tenderness and adnexal tenderness along with one of
the following:
(i) Fever >38C, (ii) Leukocytosis >10.500/cm, (iii) Adnexal
mass, (iv) Purulent material in culdocentesis, (v) purulent
16 Short Notes and Short Cases in Gynaecology

cervical discharge showing gonococci (intracellular diplo-


cocci).
3. Cervical motion tenderness as diagnostic of PID
Cervical motion tenderness only implies inflammation of pel-
vic peritoneum which occurs in many diseases other than
PID like ectopic pregnancy, appendicitis, adnexal torsion and
intraperitoneal bleed.
4. Investigations in PID
Exclude pregnancy by gravindex test
ESR, CBC, C-reactive protein
Ultrasound of pelvis
Culture of cervical discharge/Gram stain
Culdocentesis
Laparoscopy if patient does not improve within 48 to
72 hours of antibiotics.
5. Differential diagnosis of PID
Endometriosis, adnexal torsion, acute appendicitis, acute
diverticulitis, tuberculosis of genital tract ruptured ovarian
cyst, pelvic adhesions, UTI.
6. Sequelae of PID
Ectopic pregnancy, infertility, hydrosalpinx, chronic pelvic
pain tuboovarian abscess mass, dyspareunia.
One half of women with ectopic pregnancy have history
of PID. After one episode of PID 30 percent of women become
infertile.
7. Treatment of PID
Outdoor Indoor

Doxycycline 100 mg bid for Cefoxitin IV 2 g 6 hourly


for 10-14 days Plus
Cefoxitin 2 g IV + probenecid
1 gm
or
cefotetan 2 g IV 12 hourly }2 days

or ceftriaxone 250 mg IM
plus
Doxycycline 100 mg bd and
metronidazole 500 mg bid
10-14 days
Pelvic Inflammatory Disease 17

8. Hospitalization in PID
Presence of pelvic mass
Presence of IUD
Upper abdominal pain
40 certain diagnosis
Pregnancy
Non-compliance/intolerance to oral drugs
4 Abnormal
Uterine Bleeding

1. Abnormal uterine bleeding and its causes


Abnormal uterine bleeding is one that does not conform in
frequency, duration, or amount to that of the normal cyclic
withdrawal bleeding in women of reproductive age.
Causes
Complication of pregnancy (threatened abortion, ectopic
pregnancy, placenta praevia).
Malignant lesions (Leiomyoma, adenomyosis, endome-
trial/cervical polyp, uterine malignancy).
Breakthrough bleeding (HRT, contraceptives).
Coagulation disorders.
Systemic disease (hypothyroidism, hepto-renal disease).
Infections (cervicitis, endometritis, salpingitis).
Dysfunctional uterine bleeding.
2. Terms used to describe abnormal uterine bleeding
Menorrhagiaexcessive uterine bleeding at regular intervals
MetrorrhagiaUterine bleeding at irregular intervals
MenometrorrhagiaIrregular, excessive, prolonged uter-
ine bleeding
PolymenorrhoeaFrequent regular bleeding at < 18-day
intervals
OligomenorrhoeaInfrequent irregular bleeding at > 45-
day intervals
HypomenorrhoeaDecreased menstrual flow at regular
intervals.
3. Causes of menorrhagia
General diseases like blood dyscrasia i.e. thrombocy-
topenic purpura, severe anemia.
Abnormal Uterine Bleeding 19

Psychological disorders like emotional upset, matri-


monial dysharmony and anxiety state.
Local causes like fibroid uterus, salpingo-oophoritis,
endometriosis, retroversion of uterus and chocolate
cyst of ovaries.
Endocrine disturbances:
i. Early hyperthyroidsim
ii. Myxoedema
iii. Early stage of acromegaly
iv. Diabetes mellitus.
4. Polymenorrhoea or epimenorrhoea
In polymenorrhea or epimenorrhoea the menstrual cycle is
reduced from the normal twenty-eight days to a regular cycle
of two or three weeks.
5. Polymenorrhagia
This is a combined state of menorrhagia and polymenorrhoea
that is short cycle with excess bleeding.
6. Metrorrhagia
It is defined as irregular, acyclical bleeding from uterus.
7. D U B
Dysfunctional uterine bleeding is a condition of menorrhagia
in which there is no structural abnormality in pelvis or any
evidence of general or endocrine disorder. It is mostly ano-
vulatory.
8. Metropathica haemorrhagica
This is a specialised form of DUB with definite characteris-
tics. The endo-metrium of uterus is thick and polypoidal with
one or other ovary containing a cystic follicle.
It usually occurs between 40 to 45 years of age
Rarely under age of 20 and above 45
Continuous vaginal bleeding lasting for weeks
Occasionally bleeding stops and a long spell of amenor-
rhoea of about eight weeks continues
Quantity of bleeding is like normal menstrual bleeding
and painless because it is anovulatory.
20 Short Notes and Short Cases in Gynaecology

9. DUB, mechanism of excess bleeding


DUB is the most common cause of abnormal uterine bleeding
and 90 percent are anovulatory. Lack of progesterone allows
asynchronous excessive endometrial growth which is fragile
and bleeds excessively.
10. Medical conditions associated with anovulation
1. Polycystic ovarian disease
2. Pituitary tumors
3. Obesity
4. Thyroid disease
5. Hyper androgenism
6. Adrenal disease
7. Psychotropic drugs
8. Stress; physical/mental.
11. Physical findings suggestive of anovulation
Acne and hirsutism (PCO, - testosterone)
Galactorrhoea (prolactinoma, psychotropic drugs)
Thyromegaly
Obesity (PCO)
Hyperpigmentation (PCO, adrenal disease).
12. Investigation in DUB
CBC, coagulation profile, liver pannel
Prolactin, thyroid function tests
Plasma DHEA, testosterone, FSH, LH
Endometrial sampling.
13. Management of DUB
Treatment is individualised and is based on patients age
and desire for future pregnancy. In adolescents medroxy-
progesterone acetate 10 mg PO daily for first 12 days of cycle
or OCP 21 days each month produces effective control. In
reproductive age women who do not want pregnancy,
medroxyprogesterone depot 150 mg IM every three months
provides contraception and prevents menorrhagia. NSAID
also reduces menorrhagia by 50 percent. For women desiring
pregnancycromiphene or hCG may be used to induce ovu-
lation. Resistant patients may need endometrial ablation by
laser/electrocoagulation with a prior course of GnRH analog
that reduces LH, FSH and thus endometrial thickness.
5 Pap Smear

1. Pap smearClinical relevance


Pap smear is noninvasive and inexpensive outdoor office
procedure that allows early detection of cervical cancer. Its
false negative rate is 5 to 15 percent. Since natural history of
cervical cancer is 8-30 years, even with high negative rate
repeated pap smear holds key to treat, and prevent cervical
cancer. Though high risk women should be screened annu-
ally, in others screening at 1 to 3-year interval is required.
2. Risk factors for preinvasive lesions
Early intercourse
Multiple sexual partners
Early pregnancy
HPV infection
Use of oral contraceptives
Immunocompromised state
Low socio-economic status
Nicotine abuse.
3. Natural history of CINs
Cervical intraepithelial neoplasia (CIN) can progress to carci-
noma in situ depending upon the grade of CIN changes. While
regression in CIN I is 57 percent, that in CIN II is 43 percent
and that in CIN III is 32 percent. Many CIN III lesions progress
to invasive cervical cancer.
Cells from both squamocolumnar junction and endocervix
must be present for cytological examination.
4. Cellular changes in HPV infection
Cellular changes of squamous cells are characterised by
cytoplasmic vacuolization, nuclear enlargement, irregularity
22 Short Notes and Short Cases in Gynaecology

and hypochromatia. This HPV effect and CIN I change over-


lap, hence both are grouped into low grade squamous intrae-
pithelial neoplasia.
5. Predisposition to cervicitis
Multi-parity
Frequent abortion
Foreign body or cervical trauma by instruments
Gonorrhea
Poor vaginal hygiene
Hypo-oestorgenism
Vitamin deficiency.
6. Signs and symptoms of cervicitis
Leukorrhea
Infertility
Backache
Dyspareunia, dysmenorrhoea
Abdominal pain
Dysuria, frequency, urgency
Metrorrhagia
Abortion
Cervical erosion and hypertrophy.
7. Complications of cervicitis
1. Acute
Salpingitis
Peritonitis
Septicaemia.
2. Chronic
Infertility
Persistant leucorrhoea
Infection of urinary tract
Carcinoma of cervix
Arthritis
Constitutional ill health.
8. Cervical erosion and causes
It is a reddened area around the external OS.
It is more or less a misnomer because cervical erosion is
never an eroded area of cervix.
Pap Smear 23

The causes are:


Congenital erosion
Chronic cervicitis
Papillary erosion
Hyperplasia of mucous membrane of the cervix.
9. Signs and symptoms of cervical erosion
Excess vaginal discharge, non-purulent and non-
pruritic
Vaginal bleeding usually after coitus
General malaise and ill health
Remote infection like arthritis
Vague abdominal pain
Occasional urinary complaints.
10. Cervical polyps
These are red vascular swellings of size of peas or more which
bleed on touch and cause abnormal discharge or bleeding.
Endocervical polyps are red, flame-shaped and fragile growths
of about 3cms in length or less which protrude from external
OS and bleed easily.
11. Clinical features of cervical polyps
Leukorrhea
Abnormal vaginal bleeding
Sterility
Post-menopausal bleeding.
12. Treatment of cervical polyps
Culture and sensitivity of cervical mucus and appro-
priate antibiotics when infection is present
Surgical removal of polyp by avulsion, scalpel exci-
sion
Use of high frequency electro surgery
Exploration of cervix and uterine cavity for multiple
polyps
Warm acetic acid douche after polypectomy
Biopsy of tissue to exclude malignancy.
13. Cervical dysplasia
Cervical dysplasia is a pathological diagnosis made from bio-
psy of cervix. Colposcopic examination consists of painting
24 Short Notes and Short Cases in Gynaecology

of cervix with 3 percent acetic acid and then visualization


under magnification. Acetic acid has dehydrating effect in-
creasing N:C ratio and the dysplastic area becomes white
called aceto white epithelium (AWE). Vascular changes in
dysplastic area are punctations and mosaicism. Colposcopy
provides a guided approach to cervical biopsy. It takes 1 to
20 years for dysplasia to progress to invasive cancer but
some neoplasia never become invasive.
6 Uterine Leiomyomas

1. Leiomyoma and its causes


It is a benign or innocent tumor arising from muscular and
fibrous tissue of uterus. It is usually multiple, encapsulated
and sometimes attains very big size. The other name for lei-
omyoma is fibroid.
Causes
Idiopathic
Estrogen overactivity
Sterility.
2. Secondary or degenerative changes in leiomyoma
Atrophy
Hyaline degeneration
Cystic degeneration
Fatty changes
Calcareous degeneration
Red-degenerations
Sarcomatous degeneration.
3. Complications of leiomyoma
Torsion or wandering leiomyoma
Inversion of uterus
Capsule rupture
Inflammatory changes
Adnexal diseases
Lymphangiectasis and telangiectasis.
4. Clinical features of leiomyoma
Menstrual disturbances in 30 percent usually menom-
etrorrhagia
26 Short Notes and Short Cases in Gynaecology

Pressure symptom like constipation, frequency of mic-


turition and retention of urine, edema of legs and
intestinal obstruction.
Pain and feeling of weight in pelvis in one-third
Leucorrhoeal discharge
Sterility
Palpatory finding of a hard, firm, smooth and mobile
mass in pelvis.
5. Differential diagnosis of leiomyoma
Full bladder
Ovarian cyst
Pregnant uterus
Hydatidiform mole
Endometriosis and adenomyosis
Malignant ovarian tumors
Pelvic inflammatory diseases
Ectopic pregnancy
Bicornuate uterus
Sub-involution of uterus
Carcinoma of cervix and body of uterus.
6. Effects of leiomyoma on pregnancy
Abortion
Malpresentation
Placenta praevia
Premature delivery.
7. Effects of leiomyoma on labor
Uterine inertia
Obstructed labor
Delayed labor
Retained placenta
PPH.
8. Effects of leiomyoma on puerperium
Puerperal sepsis
Subinvolution
Puerperal retroversion
Secondary PPH
Inversion of uterus.
Uterine Leiomyomas 27

9. Effects of pregnancy on leiomyoma


Rapid increase in size
Increased vascularity
Injury to fibroid during labor
Inflammation, necrosis and sloughing off of submu-
cous leiomyoma
Red degeneration.
10. Factors to be considered in treatment of leiomyoma
Severity of symptoms/no symptoms
Age and parity of patient
Size of leiomyoma
Associated conditions
General condition of patient.
11. Leiomyomas and infertility
A leiomyoma can impair fertility by occlusion of cervical canal,
distortion of fallopian tubes, change or distortion of endo-
metrium that impairs implantation. They can cause spontane-
ous abortion, preterm labor and dystocia.
12. Myomas require treatment
1. Bleeding
2. Pressure
3. Rapid growth during premenopausal years or any
growth in postmenopause.
13. Medical treatment of leiomyoma
1. Depo-provera
2. GnRH analog (buserilin, naferidlin)
3. Danazol
4. RU-486; drugs only temporarily
Shrink leiomyomas that regrow on stoppage of treat-
ment.
14. Leomyomas and abnormal bleeding
Submucus leiomyomas cause bleeding due to ulceration of
endometrium on the surface. The bleeding is not due to in-
crease in endometrial area. Most likely myomas cause change
in venous drainage pattern and vascular alteration of endo-
metrial surface.
28 Short Notes and Short Cases in Gynaecology

15. Malignancy in leiomyoma


Uterine leiomyomas arise either from smooth muscle of uter-
ine blood vessels or from persistent embryonic cell nests.
Leiomyosarcomas represent 0.3 to 0.7 percent of all myomas,
are highly malignant tumors. It is not known whether myomas
undergo malignant changes or they arise denovo in myoma-
tous uterus.
16. Intravenous leiomyomatosis
It is a rare condition in which smooth muscle cells invade the
venous channels of pelvis and can extend to right heart by
contiguous growth but usually remain confined to pelvis.
7 Pelvic Relaxation

1. Uterine support
The pelvic fascia or endopelvic fascia.
Mackenrodts ligament or the cardinal ligament or the
transverse cervical ligament.
Utero-sacral ligament
Pubo-cocxygeous muscle
Ilio-cocxygeous muscle
Ischio-cocxygeous muscle.
2. Causes of prolapse uterus
Childbirth injury
Grand multiparity
Menopausal laxity of pelvic muscles
General asthenia and debility
Spina bifida occulta and split pelvis
Inadequate rehabilitation
Increased abdominal pressuse due to :
Chronic bronchitis
Obesity and constipation
Large abdominal tumor.

3. Types of prolapse
a. Anterior vaginal wall
Upper two-thirdcystocele
Lower one-thirdurethrocele.
b. Posterior vaginal wall
Upper one-thirdenterocele
Lower two-thirdrectocele.
c. Uterine descent
1st degree descent of cervix in vagina
30 Short Notes and Short Cases in Gynaecology

2nd degree descent of cervix to introitus


3rd degree descent of crevix outside introitus
Procidentiawhole of uterus outside introitus.
4. Signs and symptoms of prolapse
A sense of heaviness of dragging in the low back.
Something descending in vagina or protruding at
vulva
Reduction of protruding mass while lying down
Leucorrhoea
Low backache
Bearing down feeling over pubis
Micturition disturbance
Constipation
A firm mobile mass in lower vagina
Mild degree of vaginitis.
5. Preventive measures for prolapse
The following prophylaxis can be adopted
Ante-natal hygeine physiotherapy and exercise,
Care and management of second stage of labor
Regular emptying of bladder
A routine episitomy in all primigravida and in compli-
cated delivery
Application of low forcep if delay in second stage
Vaginal and perineal tear must be immediately sutured
properly
Post-natal exercises
Early post-natal ambulation
Adequate rest upto six months of delivery, and avoid-
ance of strenuous work
Reasonable spacing of pregnancies and less number
of pregnancies.
6. Differential diagnosis of prolapse uterus
Cyst of anterior vaginal wall
Tumor of pouch of Douglas
Myomatous polyps
Retroversion of uterus
Hypertrophic cervical elongation
Diverticula of urethra (rarely).
Pelvic Relaxation 31

7. Symptoms of cystocele
Urinary incontinence, frequency and dysuria
Bulge or lump in vagina causing mild pelvic pressure
or protruding through introitus.
8. Treatment of cystocele
Kagel exercise to strengthen pelvic floor in mild cases
Estrogen replacement
Anterior colporaphy in severe cases or failure of
above therapy in mild cases
Concomitant urethropexy if there is stress inconti-
nence as determined by urodynamic studies.
9. Vault prolapse
It is the prolapse of vaginal cuff in hysterectomised patients.
It is caused by failure of uterosacral ligaments to support the
upper vagina which hangs down causing pelvic pressure
and pain. Symptomatic patients need repair by sacrospinous
suspension (Nichols procedure) or an abdominal sacral
colpopexy suspension with a graft.
10. Colpocliesis
Colpocliesis is surgical obliteration of vagina. It is indicated
in elderly women with symptomatic prolapse who no longer
desire to preserve sexual function.
11. Pessary treatment for prolapse
It is never curative and can only be palliative.
It may cause vaginitis.
A ring type of pessary cannot be retained if vagina is
patulous.
The wearing of pessary in some women causes more
discomfort than prolapse itself.
A pressary will not cure stress incontinence.
Young women should not be forced to pessary life.
12. Types of pessary available
Ring pessary
Thomas pessary
Hodge pessary.
8 Uterine Displacement

1. Retroversion
If the cervical canal is directed upwards and backwards then
the uterus is called retroverted. In clinical practice when uterus
is not anteverted it is known as retroversion. Twenty percent
of ladies have retroverted uterus.
2. Causes of retroversion
Congenital in 20 percent cases
Intrinsic defect in myometrium
Puerperal sepsis, manipulation
Effects of prolapse uterus
Presence of tumors.
3. Clinical features of retroversion
a. Symptoms due to congestive state of uterus
Congestive dysmenorrhoea
Menorrhagia of moderate degree
Congestive state of cervical and endocervical
glands causing non-purulent leucorrhea.
b. Symptoms due to position of uterus
Dyspareunia
Sterility
Occasionally abortion
Retention of urine
A feeling like that of prolapse.
4. Fixed retroversion
It is retroversion of uterus due to some fixed adhesion usually
formed due to salpingo-oophritis, pelvic peritonitis, endo-
metriosis, pelvic tumors or chocolate cyst of ovary.
Uterine Displacements 33

5. Treatment of retroversion
Replacement of retroversion bimanually.
Pessary tretament, usually a Hodge pessary to prevent
recurrence.
Surgical treament is advised in fixed retroversion and
in case of repeated abortion.
Ventro-suspensionIn this surgical procedure two round
ligament of uterus are sutured together in front of rectus
muscle so that round ligaments are not only shortened but
attach the uterus directly to anterior abdominal wall.
6. Symptoms of retroverted gravid uterus
Backache from pressure on sacral peritoneum
Frequency of micturition
Retention of urine from stretching of urethra.
7. Treatment of retroverted gravid uterus
Bedrest with patient spending much of her, time on her stomach.
The treatment consists of introducing a self-retaining
catheter and slow catheterization of bladder. The slow emp-
tying of bladder should take about 24 hours and at the end of
this time the uterus comes to the ante-flexed position. If
adhesion persists, then operative procedure is necessary.
8. Causes of acute inversion of uterus
Acute inversion is usually puerperal
Excess traction to umbilical cord when placenta is
adherent or implanted fundally
By squeezing relaxed uterus immediately after delivery
Spontaneous, without any cause.
9. Signs and symptoms of acute inversion
Whole uterus lies outside vagina
There is severe shock
Uterus bleeds profusely.
10. Treatment for acute inversion
Treatment of shock with IV fluid specially replace-
ment by blood or plasma substitute
Anaesthetisation of patient after shock is arrested
Halothane is preferred as it causes rapid uterine relax-
ation. Tocolytics showed be given.
34 Short Notes and Short Cases in Gynaecology

Irrigation of vagina with one gallon of warm water (Ster-


ile) from a height of 3 to 4 feet after gently pushing the inverted
uterine fundus back into vagina and the vaginal orfice closed
by hands of operator.
Inversion is slowly corrected by hydrostatic pressure;
then oxytocin be given.
Manual replacement if this method fails with deep anaes-
thesia.
Abdominal total hysterectomy in old age group.
11. Causes of chronic inversion of uterus
Late puerperal cases associated with and those asso-
ciated with extrusion of a submucous myoma of
fundus
Sarcomatous myomata.
12. Clinical features of chronic inversion
Severe intermittent abdominal pain
Irregular vaginal bleeding
Profuse offensive vaginal discharge
Cup-shaped depression at fundus and inability to
palpate vaginally the portion of cervix.
13. Treatment of chronic inversion of uterus
IV fluid or blood transfusion
Digital-vaginal and uterine exploration
Complete bedrest and antibiotics
Anti-septic packing with gauze soacked with flavine
emulsion
In young, restoration of uterusby section of con-
stricting ring of cervix and then restoring fundus to
its normal place
In older age group whose uterus can be spared, vagi-
nal or abdominal total hysterectomy may be done
Myomectomy in case of myomata.
14. Aveling repositor
It is a device used for conservative treatment of chronic in-
version of uterus. It consists of a vulcanite cup placed in
contact with inverted fundus and pushed upwards by a metal
rod supported by tapes from waist bends and shoulder strap.
This is employed where no surgical facilities are available.
9 Urinary Problems

1. Genuine stress incontinence (GSI)


Stress incontinence is the involuntary loss of urine during
physical activity like coughing, laughing, sneezing, climbing.
GSI is incontinence caused by loss of normal anatomic angle
between bladder and urethra. GSI accounts for 75 to 80 per-
cent of all cases of incontinence. Common causes of GSI
include vaginal delivery, delivery of large infants, instrumental
delivery, cystocele and uterine prolapse.
2. Detrusor dyssynergia (DD)
It is called urge incontinence or unstable bladder. DD can be
caused by uninhibited bladder contraction, (motor urge in-
continence) or strong sensory input from bladder due to lack
of estrogen, inflammation and tumor. The patient cannot hold
urine once the urge is felt. DD can be due to UMN lesion as
in multiple sclerosis, where the pelvic arc is uninhibited. Par-
tial denervation of bladder after pelvic surgery can produce
DD.
3. Tests useful to differentiate GSI and DD
Cystometrogram is most helpful. It is normal in GSI
but shows reduced bladder capacity, spontaneous
bladder contractions in DD.
Cystoscopy to rule out inflammation, tumor and ana-
tomic abnormality.
Q-tip test for urethral mobility; the angle change by
more than 35 when patient coughs in GSI.
Voiding diaryIn GSI patient voids more than 6 times
per day and in excess of 250 ml each time but in DD
frequency it increase with decrease in amount.
Marshal testIn continence resolves with elevation
of urethrovesical angle in GSI but not DD.
36 Short Notes and Short Cases in Gynaecology

4. Treatment for DD
a. Drugs to enhance urine storage
Anticholinergicsoxybutynin, propantheline,
imipramine
Beta sympathomimeticsSalbutamol, orcipre-
naline
Prostaglandin inhibitorsIndomethacin
Calcium channel blockers
Musculotropic drugsFlavoxate
Dopamine agonistBromocriptine
b. Biofeedback
c. Bladder retaining technique
d. Sacral/bladder denervation.
5. Treatment of GSI
Estrogen replacement in postmenopausal women.
Kegal exercises and biofeedback.
Surgical measures to (i) restore anatomic support to
proximal urethra and urethrovesical junction in women
with hypermobility but normal internal sphincter,
(ii) procedures designed to compensate for a poorly
functioning urethral sphincter.
Teflon injection to bladder neck for its elevation and arti-
ficial sphincters are in vogue but no method is fully effective.
6. Retention of urine in Gynaecology
Operation over vaginausually for prolapse
Extended Wertheims operation
Abdominal hysterectomy
Postoperative vaginal pack
Puerperal retention may be due to vaginal tenderness
or episiotomy
Inflammatory stricture after gonorrhea
Prolapsed urethrocele
Cancer of urethra, bladder neck, vulva or vagina
Space occupying lesions of pelvis which obstruct
urethra
Hematocolpos in young girls
Retroverted gravid uterus
Pelvic hematocele
Myomas
Ovarian tumors.
Urinary Problems 37

7. Gynaecological causes of frequency of micturition


Myomatas
Ovarian cysts
Parametritis and inflammation of uterine appendages
Urethritis
8. Causes of dysuria in Gynaecology
Gonococcal urethritis
Tender caruncle at meatus
Prolapse of urethral mucous membrane
Diseases of vulva like kraurosis, carcinoma of vulva
Carcinoma of urethral meatus
Honeymoon cystitis
All operations upon urethra or near urethra
Metallic catheterization or other instrumentation in
urethra
After radium treatment of Ca-cervix or other irradia-
tion over vagina.
9. Bonneys test for stress incontinence
This is a test performed in patient undergoing surgery for
stress incontinence. The examiner places two fingers, one on
each side of urethra and exerts upward pressure against sub-
pubic angle. Then the patient is asked to cough. If urine does
not escape while pressure is maintained, it is positive indica-
tion for surgery.
10. Causes of vesicovaginal fistula
I. Gynaecological
Hysterectomy
Wertheims operation
Calporrhaphy and Manchesters operation
Radium implantation
Congenital abnormality
Bladder neck resection
Urethral diverticulum or cyst
Neglected pessary
Operation for artificial vagina
Abdominal resection of rectum.
II. Obstetric
Prolonged labor Forcep delivery
Ceserean section Rupture uterus.
10 Adnexal Mass

Adnexal masses must be diagnosed and evaluated because


of possible malignancy. Though frequently asymptomatic,
many patients have pain, dyspareunia, and tenderness.
Although majority of these neoplasms are benign and func-
tional cysts, malignancy must be excluded.
1. Differential diagnosis of adnexal mass
i. Physiologic/functionalfollicular cyst, corpus luteum
cyst, theca leutin cyst.
ii. Nonfunctionalectopic pregnancy, endometrioma,
polycystic ovary, parovarian cyst, germinal inclusion
cyst.
iii. Mechanicalovarian torsion, hydrosalpnix, tubo-
ovarian abscess.
iv. Non adnexaldeverticulitis, appendicitis, pelvic kid-
ney, leiomyoma.
v. Benign lesionBrenners tumor, mature cystic teratoma,
cyst adenoma (serous, mucinous).
vi. Malignant lesion
a. Epithelialadenocarcinoma (serous/mucinous).
b. Sex cordGranulosa cell, gynandroblastoma.
c. Germ cellDysgerminoma, choriocarcinoma, endo-
dermal sinus tumor.
2. Important patients characteristics of in adnexal mass
Malignant tumors occur in those beyond 45 years.
Germ cell tumors are common in young and 95 percent are
cystic teratoma.
Nulliparity is associated with chances of malignancy.
Physiologic cysts occur during reproductive age and have
menstrual irregularities.
Bilateral adnexal masses more likely represent malignancy.
Adnexal Mass 39

When associated with hydrothorax and ascites (Meigs syn-


drome, benign Brenners tumor is likely.
Weight loss and GI, urinary symptoms indicate possible
malignancy.
3. Characteristic of adnexal masses differentiating benign
from malignant lesions
Size : Smaller masses are more likely to be-
nign. Very large masses are likely to be
of borderline/lowgrade malignancy.
Mucinous tumors grow very large and
usually are benign.
Consistency : Cystic masses are benign but solid or
complex masses are likely to be malig-
nant.
Morphology : Benign masses are smooth with distinct
borders. Malignant masses are usually
nodular with vague borders.
Mobility : Benign masses are freely mobile but
malignant masses are non-mobile or
adherent to pelvic structures.
Number : Benign lesion is more likely to be uni-
lateral but malignant lesions are more
likely bilateral.
Ascites : It is commonly associated with malig-
nant lesions.
4. Functional cysts of ovary
Follicular, corpus luteum and thecalutein cysts (in order of
frequency). 50 percent of then regress spontaneously. Those
taking OCP do not develop cysts because of nonovulation.
5. Corpus luteum cysts
Persistent corpus luteum cysts may cause diffuse abdominal
pain, tenderness, fever and mild leukocytosis to be easily
confused with appendicitis, PID or torsion of ovary. Endocrine
effects due to progesterone secretion may lead to abnormal
uterine bleeding. If it ruptures hemoperitoneum occurs.
6. Follicular cyst
It is single, thin-walled translucent simple cyst filled with
clear to straw-colored fluid. It is innocuous but may cause
40 Short Notes and Short Cases in Gynaecology

pelvic heaviness but intraperitoneal hemorrhage after rup-


ture does not occur nor there is menstrual disturbances.
Follicular cysts in polycystic ovary are numerous, bilat-
eral, small and hormone producing arising from hyperplasia
of both granulosa and theca interna cells. Tunica albuginia is
thick and the cyst produces estrogen and androgen leading
to amenorrhoea, hirsutism and abnormal uterine bleeding.
Endometrial carcinoma is a risk since there is no production
of progesterone.
7. Thecalutein cysts
Thecalutein cysts develop with prolonged or excessive stimu-
lation of ovaries by gonadotropins. They may occur in molar
pregnancy, twins and diabetes (large placenta). Ovaries are
bilaterally enlarged but resolve spontaneously. Ovulation sup-
pression with OCP may stop further growth of cysts.
8. Endometrioma
These are endometrial implants on ovary. They are often large
bilateral multiloculated hemorrhagic cysts. Patients are usu-
ally asymptomatic but dysmenorrhea, dyspareunia and
infertility may be these. Surgical removal is needed since
malignancy is to be ruled out and there is poor response to
medical therapy.
9. Role of US in adnexal mass
US tells of size, location, consistency, complexity and rela-
tion with other pelvic organs. Ascites can be documented.
Color flow imaging can detect intraovarian vascular changes
and can discriminate benign from malignant lesions.
10. Ovarian torsion
Benign cystic ovarian mass account for most torsions. Ma-
ture cystic teratomas are the most common tumors leading to
torsion but parovarian cysts, other benign solid tumors and
serous cysts are also common. Torsion occurs twice as com-
mon with right adnexal than left.
Fever, pain, leukocytosis are the presenting symptoms
which are also shared by appendicitis, hydrosalpnix, ectopic
pregnancy, endometrioma, etc.
Adnexal Mass 41

11. CA 125
It is a tumor marker of most epithelial malignancies. Its level
can also be raised in leiomyomas, pregnancy, pelvic infec-
tions, endometriosis, and hepatitis. False negative results
may occur in 50 percent of patients with early ovarian
malignancy.
11 Ectopic Pregnancy

1. Types of ectopic pregnancy


1. Extrauterine:
Tubal
Ovarian
Primary abdominal
Cervical
2. Uterine:
Interstitial pregnancy
Pregnancy in an accessory cornu in a bi-cornuate
uterus.
2. Causes of ectopic pregnancy
a. Inflammatory
Catarrhal salpingitisfollowing puerperal sepsis
Endosalpingitis
Tuberculous salpingitis
Gonococcal salpingitis
Extragenital pelvic infection like pelvic abscess,
chronic appendicitis.
b. Congenital anomalies
Congenital defect of fallopian tube likeacces-
sory ostia, diverticulum, partial stenosis and hypo-
plasia and undue tortuosity of tube.
c. Pelvic abnormalities like:
Fundal myomata
Adenomyoma of fallopian tube.
d. Certain pelvic operations like ventrosuspension.
e. Defect with ovum itself like rapid or slow develop-
ment of trophoblast.
Ectopic Pregnancy 43

f. Increase receptiveness of mucosa of fallopian tube


to hormone of ovary.
g. IUD use DES exposure, progestin only contraceptive.
3. Complications of ectopic pregnancy
Tubal mole formation
Tubal abortion
Tubal rupture
Peritubal hematocele
Pelvic hematocele
Internal tubal hemorrhage
Broad ligament hematoma
Paratubal hematocele
Diffuse intraperitoneal bleeding
Spontaneous regression
Secondary abdominal pregnancy.
4. Fate of ovum in ectopic pregnancy
Reabsortion of ovum during early pregnancy
Mumification in advanced stage of pregnancy
Calcification and formation of lithopedion
Delivered alive.
5. Symptoms of ectopic gestation
History of pregnancy or amenorrhea.
Painusually abdominal and constant in most of
cases.
Hemorrhagic shock when tubal pregnancy ruptures,
shoulder pain in 25 percent due to diaphragmatic irri-
tation.
Vaginal bleedingaltered or dark fluid blood per
vagina in a continuous manner.
Passage of decidual cast is pathognomonic of ectopic
pregnancy.
6. Physical signs of ectopic pregnancy
Signs of early pregnancy in general
Abdominal examination reveals nothing.
Tenderness in lower abdomen in 90 percent cases.
Abdominal distension may be due to presence of free
blood in peritoneal cavity.
44 Short Notes and Short Cases in Gynaecology

Cullens signin repeated intraperitoneal bleeding for


more than two to three weeks, umbilicus may show blu-
ish discoloration.
Tenderness of cervix during examination is an important
sign.
Palpable pelvic mass in 50 percent cases; 20 percent have
ectopic contralateral to mass.
7. Differential diagnosis for ectopic pregnancy
Threatened or incomplete abortion
Retroverted gravid uterus
Pelvic inflammatory conditions, usually a pyosalpinx
DUB degenerating leiomyoma
Torsion of ovarian cyst/adnexa
Endometriosis
Hemorrhage from corpus luteum
Non genital conditions like: Acute appendicitis, ure-
teric colic, diverticulitis and cystitis.
8. Risk of ectopic pregnancy according to contraceptive
methods
Contraception Risk of ectopic %
None 1
OCP 1
IUD 5
Progestasert IUD 15
9. Recurrence risk of ectopic pregnancy
7-15 percent.
10. Diagnosis of ectopic pregnancy
Symptoms and signs
HCG but subnormal for duration of pregnancy
US (demonstration of fetal heart activity most defini-
tive)
Culdocentesis (blood that does not clot)
Laparoscopy.
11. Treatment for ectopic pregnancy
If there is rupture and diffuse intraperitoneal bleed-
ing then patient is immediately treated for shock and
Ectopic Pregnancy 45

hemorrhage along with arrangements for immediate


laparotomy and excision of fallopian tube with ectopic
mass.
If hemorrhage is localized and there is no immediate
danger, an arranged laparotomy is performed.
In case of abdominal pregnancy in early stage of preg-
nancy laparotomy is immediately done and the small
fetus is removed. Placenta is not removed due to risk
of severe hemorrhage. It is left in abdomen which
disintegrates in 1 to 2 years. A course of methotrex-
ate may cause quick disintegration of placenta.
In case of a living fetus or dead fetus the laparotomy
may be delayed.
12. Role of methotrexate and actinomycin D
Single 50 mg/m2 IM dose has success rate of 95 percent
with tubal patency preserved in 80 percent. It is very helpful
in treating interstitial, abdominal and cervical pregnancy
which have greater surgical risk. When hCG level is more
than 10,000 unit methotrexate has higher failure rate and may
demand actinomycin. D. KCl injection to fetal heart to induce
asystole is an alternative.
12 Endometriosis

1. Endometriosis
Endometriosis is the presence of tissue similar to normal
endometrium in structure and function at sites other than
lining of uterine cavity. The most common sites are anterior
and posterior cul-de-sac, ovaries (in 65%), broad ligament
fallopian tubes.
2. Aetiological theories for endometriosis
Implantation theory
Metaplasia theory
Lymphatic theory
Blood-borne theory
3. Microscopic appearance of endometriosis
Presence of endometrial glands and stromal tissue are char-
acteristic. These glands show periodic changes under
influence of ovarian hormones like endometrium of uterine
cavity. So during menstruation, the ectopic endo-metrial
tissue shows proliferation and bleeds like uterine endometrium.
But as there is no outlet for blood, it forms a cyst at the site.
The retained blood turns chocolate color.
4. Presenting symptoms of endometriosis
Asymptomaticif the lesion is very small
MenorrhagiaIt is a common symptom and is seen
in endometriosis of pelvic site
Dysmenorrhoea (25-50%) The pain starts on 1st day
of menstruation or may be prior. During menstrua-
tion, the pain is localized to the site of endometriosis
Dyspareuniadifficulty during coitus is common
with endometriosis of recto-vaginal septum and pouch
of Doughlas
Endometriosis 47

Sterility (30-40%) endometriosis may cause sterility


but actual scientific basic is not yet cleared.
Acute abdomensometimes the leakage or rupture
of endometriosis cyst may cause acute severe pain
due to peritonitis giving rise to symptoms of acute
abdominal emergency.
5. Physical signs of endometriosis
No clinical sign in very small lesion.
Nodular masswhen cysts are formed, nodular mass
can be felt on surface of organ, in adnexa, (often asym-
metric)
Retroversion of uterusin case of endometriosis of
pouch of Doughlas uterus is usually retro-verted and
fixed.
TendernessTenderness is felt during pelvic exami-
nation or pain is felt at lower abdomen during men-
struation and is diffuse.
Bluish nodulesin endometriosis of umbilicus,
nodular thickening and tenderness along utero sacral
ligament, posterior surface of uterus and cul-de-sac
scarring and narrowing of posterior vaginal fornix.
6. Common sites of endometriosis
The common sites in order of occurrence
Ovaries80 percent
Pelvic peritoneum
Large and small intestines
Cervix, vagina and vulva
Urinary tract
Umbilicus and abdominal scars
Limbs, groin and lungs.
7. Appearance of endometriosis
Powderburn appearance
Haemorrhagic or flame shaped lesion
Fibrotic-scarred peritoneum
Cystic or vesicular
Chocolate cyst (ovary).
8. Line of treatment for ovarian endometriosis
Expectant treatment
48 Short Notes and Short Cases in Gynaecology

Hormonal treatment
Irradiation menopause
Surgery
9. Medical treatment of endometriosis
1. Oral contraceptives (estrogen-progesterone) to in-
duce pseudopregnancy state, so that the implant
undergoes necrosis and is absorbed.
2. Danazolmost useful drug that suppresses gonado-
tropin release, inhibits steroidogenesis and interferes
with cytosolic hormone receptors.
3. Medroxy progesteronea potent gonadotropin in-
hibitor
4. GnRH analogsthey do not improve fertility but
reduce pain by exhaus-tion of gonadotropin release.
10. Pathogenesis of endometriosis interna
It involves usually the posterior uterine wall.
It may be localized or diffuse
In diffuse type it causes uniform enlargement of uterus
On section of uterine wall multiple small cystic areas are
seen. Chocolate colour altered blood may be present in
the cysts
Unlike externa, the response of ovarian hormones to this
internal tissue is very minimal.
11. Symptoms and signs of endometriosis interna
Menorrhagia
Dysmenorrhoea
Frequency of micturition.
Uniform enlargement of uterus
Tenderness on bi-manual examination.
12. Stromal endometriosis
It is a myometrial tumour composed of endometrial stromal
cells. Nowadays it is considered as a low grade sarcoma.
13. Pathological feature of stromal endometriosis
The uterus is enlarged
The cut surface shows localized protruding worm like
masses
Such masses extend to uterine cavity or broad ligament
Endometriosis 49

Histologically round and oval cells of endometrium


are seen.
14. Clinical features of stromal endometriosis
Menorrhagia
Pain in pelvic region
Lower abdominal swelling.
15. Treatment for stromal endometriosis
Hysterectomy with bilateral oophorectomy
The ovaries may be preserved in young patients, and
Radiotherapy for local recurrence and for incomplete
surgical removal.
16. Chocolate cyst of ovary
The affected ovary is enlarged and its outer surface
is white and thickened.
The ovary and fallopian tubes are prolapsed and fixed
to the back of uterus or to broad ligament
Usually the cyst measures about an inch in diameter
or less
It is bilateral in one-third cases
There is thickening of tunica albuginea
The cyst fluid is thick and chocolate like in colour
due to altered blood. The chocolate cyst of ovary is
the same as ovarian endometriosis. The other descrip-
tions like signs and symptoms differential diagnosis,
treatment, etc. have been dealt earlier.
17. Common sites of intestinal endometriosis
The common sites are:
Sigmoid colon, rectum 72.4 %
Recto-vaginal septum 13.5 %
Small intestine 7%
Caecum 3.6 %
Appendix 3%
Elsewhere 0.5 %
18. Symptoms and signs of intestinal endometriosis
Mild weight loss
Constipation associated with menses
50 Short Notes and Short Cases in Gynaecology

Bleeding on defaecation during menses


Fertility decreased
Dysmenorrhoeafrequent and progressive
SigmoidoscopyIntact mucosa
Barium enemaLong filling defect
Biopsytypical endometrial features
Relatively mobile tumour
No enlargement of mesenteric glands
19. Line of treatment of intestinal endometriosis
If patient is young and obstructive features are
present, resection of affected segment of intestine
If patient is about 40 with intestinal obstruction
Resection of bowl with hysterectomy and bilateral
salpingo-oophorectomy
If lesion is present but no obstruction in young
patient, conservative surgery of ovarian endometrio-
sis. Oral progesterone is helpful in this case
If patient is over 40 and without obstructive feature
then total hysterectomy with bilateral salpingo-
oophorectomy is done.
13 Acute and Chronic
Pelvic Pain

1. Chronic pelvic pain


Chronic pelvic pain in women is nonspecific pain of more
than 6 months duration, not relieved by narcotic analgesics.
It accounts for 10-35 percent of laparoscopy and 10 percent
of hysterectomies in West. The structural damage lags behind
the degree of pain.
2. Innervation of pelvic organs
The innervation of lower abdominal wall, and anterior aspect
of vulva including clitoris and urethra is from L1L2 which also
receives sensation from lower back. The perineum, anus and
lower vagina is supplied by S2S3S4. Painful stimuli from upper
vagina, cervix, uterus, fallopian tubes, bladder, cecum,
appendix and terminal large bowel travel in thoracolumbar
sympathetics, and then via white rami communicates to enter
spinal cord at T11 T12 L1 segment. Since rectum, bladder and
part of vagina are embryologically derived from urogenital
sinus, they have sacral afferents in addition to thoracolum-
bar afferents. The afferent pathway from ovary, upper ureter,
outer two-thirds of fallopian tube enterspinal cord at level of
T9 T10 segments.
3. Differential diagnosis of chronic pelvic pain
Gynaecological Others
Pelvic inflammatory disease Stress fracture pelvis
Endometriosis Interstitial cystitis
Pelvic adhesion Irritable bowel
syndrome
Pelvic relaxation Bladder spasm
Pelvic congestion Diverticulitis
52 Short Notes and Short Cases in Gynaecology

Ovarian cyst Chr. appendicitis


Inflammatory bowel
disease
4. Causes of acute pelvic pain
1. Mittelschmerz
2. Functional ovarian cysts (follicular, corpus luteum
cysts)
3. Ectopic pregnancy
4. Pelvic inflammatory disease
5. Degenerating/torsion of leiomyoma
6. Adnexal/ovarian torsion
7. Endometriosis
8. Appendicitis, diverticulitis, strangulated hernias.
5. Carnetts test
It differentiates abdominal wall pain from visceral pain. In this
test the rectus muscles are tensed. Abdominal wall pain wors-
ens. Patients with visceral pain often present with tense
rectus abdominis.
6. Injection treatmentdiagnostic and therapeutic value
Often levator muscle spasms is source of pain, which can be
diagnosed by palpation. Injection of trigger area is of diagno-
stic as well as therapeutic value. Trigger points in paracervical
region or margins of vaginal cuff occur after hysterectomy.
Their injection with procaine/bupivacaine relieves pain.
7. Myofascial trigger pointsdetection and treatment
These are hyper irritable spots within a taut band of skeletal
muscle or fascia which are painful on compression often
causing referred pain and autonomic phenomenon (tearing,
tinnitus visual disturbances). Their treatment involves local
anesthetic injection, TENS, acupuncture.
8. Pelvic vascular congestion syndrome
This pain is worse premenstrually after standing and after
coitus. Laparoscopy discovers pelvic varicosity and pain
pathology is similar to migraine headache.
9. Premenstrual syndrome (PMS)
Physical symptoms: Weight gain, breast tenderness, acne,
hot flashes, headache, carving for sweets, pelvic pain.
Acute and Chronic Pelvic Pain 53

Emotional symptoms: Insomnia, irritability, anxiety,


depression, fatigue.
10. Treatment options for PMS
Anxiety Alprazolam
Breast tenderness Bromocriptine
Edema, bloating Spironolactone
Tension, depression Antiprostaglandins
Irritability Danazol
Measures to increase serotonin level are helpful. Vitamin
B6 200 to 800 mg/day reduces estrogen and increases proges-
terone. Vit E, calcium, magnesium may increase serotonin level
and help to relieve symptoms of anxiety, fluid retention and
insomnia.
14 Contraception

1. Mechanism of action of oral contraceptives


1. Estrogen effectovulation prevention by inhibiting
FSH.
2. Progesterone effectovulation prevention by inhi-
bition of LH, product; on of thick viscid scanty cervi-
cal mucus preventing sperm penetration.
3. Altered motility of uterus and fallopian tubes.
4. Alteration of endometrium by decreasing glycogen
production.
2. Failure rate of contraception
Sterilization 0.3/100 women per year
OCP (combination) 0.16-0.30
Norplant 0.8
Depo provera 00-0.7
IUD 1.2-3
Diaphragm 2/100
Condoms 3.5-4
3. Absolute contraindications for OCP use
History or presence of thrombophlebitis, thromboem-
bolic disease or cerebrovascular disease
Significantly impaired liver function
Known or suspected cancer of breast
Undiagnosed abnormal vaginal bleeding
Known or suspected pregnancy
Smokers above age of 35 years.
4. Association of oral contraceptives with cancer
1. Endometrial cancerRisk reduced by 50 percent
with OCP use, greatest effects seen 3 or more years
Contraception 55

after use; protection persists for more than 15 years


after discontinuation.
2. Ovarian cancerRisk of epithelial ovarian cancer
reduced by 40 percent (11% per year of use); 80 per-
cent risk reduction after 10 or more years of use.
3. Breast cancerNo effect or decreased incidence.
5. Noncontraceptive benefits of OCP
Decreased incidence benign breast disease and PID
Improvement in menorrhagia and dysmenorrhea
Improvement in endometriosis
in bone density, protection against atherosclerosis
Improvement in symptoms of rheumatoid arthritis
incidence of functional ovarian cysts
Decreased risk of ovarian and endometrial cancer.
6. Lipoproteins and OCP
Estrogens increase HDL, progestins HDL and LDL. Newer
progestins (norgestimate and disogestrol) increase HDL.
7. Depoprovera contraception
150 mg IM provides contraception for 3 to 4 months by ovu-
lation inhibition. Failure rate is minimal. Irregular bleeding
and spotting, headache, bloating and hairloss are common.
Return of fertility may take 18 months to return. On contrary
norplant provides contraception for 5 years and return of
fertility is rapid.
8. Postcoital contraception (morning after pill)
Ovral 2 pills twice daily 1 day
Ethinyl estradiol 2.5 mg PO bid 5 days
Conjugated estrogen 15 mg bid PO 5 days
Failure rate ranges from 0.1 percent with ethinyl estradiol
to 2 percent with OCP. Treatment should begin within 72
hours. If pregnancy occurs, termination should be advised.
9. Best candidates for IUD
Older parous women who are not willing for tubectomy. They
should not have multiple sex partners, history of PID or
ectopic pregnancy.
56 Short Notes and Short Cases in Gynaecology

10. Mechanism of action of IUD


Interference with sperm transport
Inhibition of sperm capacitation and survival
Endometrial changes that inhibit implantation.
11. Contraindications for IUD
Confirmed or suspected pregnancy
Suspected pelvic malignancy
Undiagnosed vaginal bleeding
Pelvic infection
Polygamous behavior.
15 Abortion

1. Abortion/miscarriage
Termination of pregnancy whether spontaneous or induced
before 28 weeks of gestation is called abortion. 15 to 20 per-
cent of pregnancies end up in miscarriage but embryonic
loss reaches upto 31 percent. Approximately 20 percent of
women bleed in first trimester of pregnancy and one half to
two-third of them abort. There is also a greater risk for subse-
quent preterm delivery and low birth weight infants.
2. Types of abortions
Threatened abortion
Inevitable abortion
Complete abortion
Incomplete abortion
Missed abortion
Recurrent or habitual abortion
Septic abortion
Therapeutic abortion
Criminal abortion.
3. Causes of abortion
A. General causes:
1. Acute illness (i) High fever and toxic conditions
like measles, scarlet fever, cholera, enteric fever,
diphtheria, pneumonia, malaria, erysipelas and
other acute septicemic or toxemic conditions.
2. Chronic medical diseases like:
(i) Diabetes mellitus (ii) syphilis (iii) jaundice
(iv) Graves disease (v) Brights disease (vi) chronic
renal failure (vii) chronic pyelonephritis (viii) thy-
roid malfunction (ix) severe anoxia.
58 Short Notes and Short Cases in Gynaecology

3. Psychogenic:
Sudden emotional outburst like sudden fear, bad
news or good news.
4. Nutritional deficiency:
(i) folic acid deficiency, (ii) avitaminosis (iii) mul-
tiple dietary deficiency.
5. Drugs consumed by mother:
Strong purgatives, quinine, lead, phosphorus,
ergot preparation, mercury preparation, metronida-
zole
B. Local causes
1. Ovaries:
Corpus luteum deficiency.
2. Uterus:
Congenital defects, displacements, neoplastic, sur-
gical operation, placenta praevia.
3. Fetal causes:
Genetic or chromosomal abnormality, maternal
irradiation, impaired placental circulation.
4. Fault of Male:
Undue pressure over abdomen during intercourse,
frequent intercourse during 1st trimester causing
excess stimulation of uterus, fault with semen
5. Local injuries:
Injury to vulva, attempt for criminal abortion by
local insertion of some object.
6. Physical strain:
Violent physical exercises, jumping, skiping, slid-
ing, long journey by bus, train or even air, by two
wheeler and three wheeler, short journey by
rickshaw or other mode of transport having no
shock-absorber device, swimming and diving.
4. Mechanism of abortion
Expulsion of the whole ovum in one piece
Expulsion of whole ovum by inversion of decidua
vera
Incomplete expulsion of the ovum-placenta and mem-
branes being retained.
Abortion 59

5. Complications of abortion in general


Hemorrhage
Retention of placenta or ovum or membrane leading
to incomplete abortion
Blockage of Fallopian tube causing secondary sterility
Septic condition.
6. Threatened abortion
Threatened abortion means there is threat or fear for abortion,
or the process of abortion has started but it can be controlled
or arrested if proper remedies are taken. In other words there
is possibility of continuance of pregnancy in threatened abor-
tion.
7. Clinical features of threatened abortion
Bleeding per vagina usually small in quantity; may increase
later on.
The blood is bright red
May be associated with clots
No pain but some discomfort in lower abdomen
On P/V examination OS is closed
Size of uterus corresponds to the size of gestation.
8. Differential diagnosis of threatened abortion
Missed abortion
Ectopic pregnancy
Metropathia hemorrhagica
Cervical mucous polyp
Early carcinoma of cervix.
9. Investigation of threatened abortion
Pelvic examination, if internal OS is closed
Estimation of pregnanediol and hCG
HCG level should double every 2 to 3 days in normal
viable pregnancy for first 7 to 8 weeks
Transvaginal US for fetal heart activity and growth.
10. Treatment of threatened abortion
Complete bedrest (is not essential)
Sedatives
Easily digestible diet
60 Short Notes and Short Cases in Gynaecology

Treatment of cause
i. In hormone deficiency, hormonal therapy
ii. Treatment of retroversion and uterovaginal pro-
lapse
Restriction for purgatives, tampoons, douching
Avoidance of hot bath
Avoidance of per vaginal or abdominal examination
unless it is essentially necessary
Weekly intramuscular injection of hydroxy progest-
erone caproate in oil upto 18 to 20 weeks
Injection progesterone 25 mg aqueous solution IM
twice a week
Nor-ethisterone, Dimethiseterone or Norethinodrel
tab 20 to 30 mg daily orally
Allylestrenol tab 5 mg thrice daily
Chorionic gonadotropin 1000 to 2000 IU weekly.
11. Outcome in threatened abortion
Immediate abortion 25 percent
Late abortion 4.5 percent
Continuance of pregnancy beyond 28 weeks with
delivery of healthy baby 70 percent.
12. Inevitable abortion
Inevitable abortion means that the abortion is inevitable or it
must occur. The process of expulsion of product of concep-
tion has not started but it will occur and no method can stop
this process.
13. Clinical features of inevitable abortion
Bleeding per vagina is initially slight and bright red
and later becomes severe with clots
From moderate pain to severe agonizing pain due to
severe uterine contraction like labor pain
Size of uterus corresponds to the size of gestation
OS starts dialating unlike threatened abortion where
OS is closed. This is a pathognomonic sign of inevi-
table abortion.
14. Treatment
1. Below 12 weeks of pregnancy immediate dilatation
and curettage is done under general anesthesia.
Abortion 61

2. Above 12 weeks IV drips of glucose with 5-10 units


of oxytocin to induce uterine contraction. This is fol-
lowed by D and C, supported by prostoglandin
carboprost 250 g IM or dinoprostone 0.5 mg endo-
cervical or 0.5 mg so hourly for 4 doses.
15. Incomplete abortion
When some part of conception is retained in uterine cavity
after abortion, it is called incomplete abortion. Usually the
piece is of placenta or chorion.
16. Signs and symptoms of incomplete abortion
History of expulsion of product of conception recently
Persistent vaginal bleeding, may be oozing or moderate
bleeding for days or months
Regular menstruation is not established
Intermittent colicky low abdominal pain or dull aching
Size of uterus remains normal or some times enlarged,
subinvolution is one important sign though consis-
tency changes
On per vaginal examination OS is always found
dilated and cervix patulous.
17. Clinical features of complete abortion
History of complete expulsion of product of conception
Relief from pain and bleeding stops within 8 to 10
days
Size of uterus comes to normal or in involution phase;
OS is closed
The regular menstruation starts within 1 to 2 months.
18. Missed abortion
It denotes pregnancy that fails to progress and the ovum
dies. This can be known from failure of hCG to rise propor-
tionate to gestation, failure of increase in uterine volume and
lack of fetal activity in transvaginal US. Hypofibrinogenemia
with DIC is the most feared complication.
19. Recurrent abortion (recurrent pregnancy lossRPL)
RPL is defined as three or more consecutive spontaneous
abortions. After 2 consecutive miscarriages the chance of
62 Short Notes and Short Cases in Gynaecology

live birth is 70 to 75 percent and after three losses it is 50 to 65


percent.
20. Rh isoimmunisation following abortion
Any Rh negative patient with Rh positive husband having
spontaneous abortion or ectopic pregnancy at less than 12
weeks should take 50 g of anti D. Beyond 12 weeks she
should receive full dose of 300 g.
21. Evaluation of RPL
TORCHS
ANF, antiphospholipid titre
GTT
T3T4TSH
Grouping, RH typing
Hysterosalpingography
US pelvis
Vaginal cytology
Serum progesteron 1 week after ovulation less than
10 ng/ml indicates corpus luteum deficiency
Chromosomal analysis of husband and wife.
22. Common correctible uterine anomalies in RPL
Septate uterus is the most commonly reported anomaly
and hysteroscopic removal of septa results in preg-
nancy in more than 80 percent cases.
Other correctible anomalies are leiomyoma, Asherman
syndrome, incompetent cervix, retroversion.
23. Methods of vaginal evacuation in relation to weeks of
gestation
Upto 10 weeks MR syringing
More than 10 weeks Suction curretage after laminaria
tent dilatation
13-16 weeks Dilatation with multiple lami-
naria, then evacuation
17-24 weeks Prostaglandin suppository, dila-
tation and evacuation.
Carboprost 250 g IM, Dinoprostone 0.5 mg endo-
cervically or 0.5 mg tab orally hourly (m m 4 tab).
Abortion 63

24. Causes of recurrent pregnancy loss (RPL)


General An identifiable cause can be found in 50 to 60
percent cases.
Infection like syphilis, TORCH
Metabolic disease like diabetes
Renal causes like chronic renal failure and pyelone-
phritis
Maternal smoking, alcohol
Endocrine disease like hypothyroidism
Alloimmunity
Nutritional deficiency like folic acid deficiency.
Lupus anticoagulant, antiphospholipid.
Local causes
Ovarian cause like corpus luteum deficiency
Uterine causes like
Congenial malformation (unicornate, bicornuate
Ashermans syndrome)
Cervical incompetency
Uterine displacements
Neoplastic leisons leiomyoma.
Foetal causes
Genetic causes
Congenital malformation
25. Shirodkars operation
It is done in cervical incompetency causing RPL. The cervix
is drawn downwards with a volsellum. A small incision is
made in the anterior fornix and another in the posterior fornix.
An aneurism needle is passed from the posterior incision
round the cervix so that its point comes out in the anterior
incision where it is threaded in the anterior incision with a
ligature. When withdrawn through the posterior incision one
half of the cervix is circumnavigated by the ligature. The simi-
lar procedure is done on other half of cervix to completely tie
with requisite tension to effectively close the incompetent
canal.
26. Septic abortion
Usually the main cause of septic abortion is the faulty aseptic
procedure adopted during criminal abortions. When solution
64 Short Notes and Short Cases in Gynaecology

or instruments or other abortifisants are introduced in cavity


without proper sterilization, infective organisms are intro-
duced to the eroded mucous surface causing sepsis.
27. Organisms responsible for septic abortion
E Coli, Hemolytic and non-hemolytic streptococci, sta-
phylococus aureus, anerobic streptococus, gonococcus,
pneumococcus, clostridia welchi
28. Clinical features of septic abortion
History of abortion, usually incomplete
Toxic look
Pyrexia of moderate or severe degree
Tenderness on palpation of lower abdomen
Foul blood stained discharge per vagina
Tenderness during P/V examination
Sometimes patient in shock with oliguria
29. Treatment for septic abortion
Immediate high vaginal swab, culture and sensitivity
blood transfusion/IV fluid
Administration of antibiotics IM/IV immediately
before result of culture and sensitivity is obtained
Appropriate antibiotics after result of culture and sen-
sitivity is obtained.
When the infection is in control (usually after 5 to 8
days and patients general condition improves, then
exploration of uterine cavity to remove product of
conception if any.
30. Complications of septic abortion
a. Immediate
Bacteriaemic shock
Anuria
Acute peritonitis if there is perforation of uterus.
b. Late
Parametritis
Thrombo-phlebitis
White leg
Salpingitis
Tubo-ovarian abscess
Generalized septicemia
Abortion 65

31. Dangers of criminal abortion


Vaso vagal shock
Air embolism
Perforation of uterus and its complications
Injury and laceration of vagina and cervix
Sepsis and peritonitis
Hemorrhage and renal failure
32. Indications of therapeutic abortion
Social where liberal law exists
Psychological in psychic patient
Medical
i. Maternal (a) general disease like heart disease,
neoplastic, hypertension, renal and respiratory dis-
eases.
ii. Complication of pregnancy like hydatidiform
mole, pre-eclampsia and liver necrosis
iii. Foetal(a) Suspicion of mal-formatioan of baby
(b) Genetic diseases (c) Rh incompatibility (d) Viral
infection of mother.
33. Size of karmal cannula for MR syringing
4 mm cannula
for primigravida
for 10 days of missed period
for relaxation of cervix
5 mm cannula for 15 to 20 days missed period
6 mm cannula for 20 to 30 days missed period
7 and 8 mm cannula used if more foetal tissue than
expected.
34. When will you feel that the procedure is complete?
Small bubbles or red foam will be seen-in the cannula
and no tissue will come out.
Typical gritty feeling of the cannula against the evacu-
ated uterine wall.
A strong uterine contraction will start if uterus is com-
pletely evacuated and the cannula will be firmly
grasped at internal OS making continued aspiration
difficult.
66 Short Notes and Short Cases in Gynaecology

35. MTP act


It is an act to provide for the termination of certain pregnan-
cies by registered medical practitioners and for matters
connected therewith or incidental thereto. MTP act was intro-
duced in August 1971.
By one registered medical practitioners if pregnancy is
less than 12 weeks
By two registered medical practitioner if pregnancy is
more than 12 weeks but less than 20 weeks. (maximum
limit for MTP permissible under MTP Act is 20 weeks).
36. Indication for MTP under MTP act
The continuance of pregnancy would involve a risk to the
life of the pregnant woman or of grave injury to her physical
or mental health.
There is substantial risk that if the child was born, it would
suffer from such physical or mental abnormalities as to be
seriously handicapped.
16 Vulva, Vagina, Cervix

1. Common causes of vulval swellings


Vulval cysts like sebaceous cyst, epidermal cyst,
Gartners duct cyst, Bartholins cyst.
Varicosity in vulva and hydrocele of canal of Nuck.
Neoplastic swellings.
2. Neoplastic tumors of vulva
1. Benign tumors
Papilloma
Fibroma and fibromyoma
Neurofibroma
Lipoma
Adenoma of Bartholins gland.
2. Malignant tumors
Carcinoma
Basal cell carcinoma
Sarcoma
Melanoma
Adeno-carcinoma of Bartholins gland
Secondary neoplasms.
3. Common vaginal swellings
1. Cysts like
Gartners duct cyst
Epidermoid cyst
Endometrioisis
Cysts of Skenes tubules
Diverticulum of urethra.
2. Neoplasms
Benign-like papilloma, fibroma, lipoma, myoma
Malignant: carcinoma, sarcoma, mixed mesoder-
mal tumors and secondaries.
68 Short Notes and Short Cases in Gynaecology

4. Varicosity of vulva
Varicosity of vulval veins are usually seen during pregnancy
due to pressure of gravid uterus, so it disappears after delivery
of baby. Minor varicosity does not cause problem. Severe
varicosity causes discomfort. The only treatment is rest and
elevation of leg.
5. Sebaceous cysts of vulva
Sebaceous glands are present in vulval skin like other parts
of body. The ducts of sebaceous glands sometimes get
blocked and the secretion of gland is accumulated to form a
cyst, giving a palpable vulval swelling. Sometimes, multiple
sebaceous cysts are found. The cysts contain thick cheesy
material. They usually do not cause any symptoms until they
are infected when excision is the only treatment.
6. Epidermoid cyst of vulva and vagina
These cysts arise due to implantation of stratified squamous
epithelium into deeper tissue. The process may occur follow-
ing birth trauma or operation on vulva. The cysts are small in
size varying from few milimeters and contains sebaceous
material. Mostly the epidermoid cysts are found in perineal
area following episiotomy operation. Usually the cysts do
not cause any symptoms. If any discomfort, excision is the
line of treatment.
7. Gartners duct cyst
During intrauterine life, a portion of main mesonephric duct
may persist as Gartners duct. This duct may lie in broad
ligament, wall of body of uterus, cervix, anterolateral wall of
vagina and labia majora. Sometimes a cyst is formed and is
known as Gartners duct cyst. The wall of cyst is lined by
cuboidal or columnar epithelium. The cyst does not cause
any symptom except the feeling of swelling. A small cyst is
treated by excision whereas a large or pedunculated cyst is
treated by marsupialization.
8. Hydrocele of canal of nuck
This hydrocele is formed due to accumulation of fluid in the
remnant of peritoneal covering of the round ligament when it
passes through the inguinal canal to labium majus. Sometime
Vulva, Vagina, Cervix 69

it is misdiagnosed as inguinal hernia. The hernia is usually


reducible whereas the hydrocele is irreducible cystic swell-
ing. The treatment is surgical excision.
9. Hidradenoma
This is a benign small solid tumor of sweat glands. Sweat
glands are present in skin of vulva. So this tumor arises in
anterior part of vulva. Sometimes the overlying skin becomes
red and edematous and occasionally it may ulcerate. The
tumor sometimes is mistaken for carcinoma of vulva. But
biopsy confirms diagnosis. The treatment is by excision.
10. Vulvar intraepithelial neoplasia (VIN)
Itching is the most common symptom of VIN. Lesions may be
white, red, pigmented, flat or raised; hence biopsy any abnor-
mal looking area be done. Anal canal also be evaluated for
VIN, since half have anal canal involvement.
VIN has been known as Bowens disease or bowenoid
papulosis. 90 percent VIN lesions have HPV DNA. Treatment
of VIN is by laser ablation, 5 percent 5FU, and skinning vul-
vectomy with skin graft. Biopsy be done every 6 months to
rule out recurrence.
11. Vaginal adenosis
Vaginal adenosis is the presence of glandular epithelium in
vagina and its mucinous secretion into vagina. This is due to
prenatal exposure to DES. It does not progress to cancer but
can be associated clear cell adenocarcinoma.
12. Vaginal intraepithelial neoplasia (VAIN)
Fifty percent of women with VAIN have neoplasia at other
sites in the lower female genital tract usually involving vulva
and cervix. VAIN is screened with pap smear and diagnosed
with colposcopically directed biopsies. VAIN usually affects
upper third of vagina and with Lugols iodine take lighter
color than the brown color given by normal vaginal cells due
to higher glycogen content. Patients are usually asympto-
matic or have post coital bleeding. HPV, radiation and immuno-
supression increase development of VAIN. Treatment is with
topical 5 Fu (85% success), laser ablation (70-80% success)
and surgical excision (80% success).
70 Short Notes and Short Cases in Gynaecology

13. Cervical intraepithelial neoplasia (CIN)


CIN defines a continuum of abnormalities seen within the
epithelium of the cervix. Normally an orderly progression of
maturation extends from the basement membrane to the epi-
thelial surface layer. A pathologic diagnosis of CIN is made
when the polarity of cellular differentiation is lost and the
cells become large with pleomorphic nuclei and scant cyto-
plasm. A crowding of cell is seen with increase in mitotic
number and nuclear to cytoplasmic ratio. Treatment of CIN
is laser cone biopsy, loop electrosurgical procedure (LEEP),
and cold knife cone (CKC) biopsy. After treatment Pap smear
be performed every 3-4 months for one years; if negative
cure may be presumed.
14. Indications for conebiopsy of cervix
1. Dysplastic cells in endocervix.
2. Evaluation of microinvasion in cervical cancer.
3. An unsatisfactory colposcopy.
4. A two degree difference of abnormality between pap
smear and cervical biopsies.
15. Koilocytes
Koilocytes are found in epithelial layer, have atleast one to
two of the following characteristics. (i) pyknotic raisinoid
nucleus, (2) perinuclear cytoplasmic halo, (iii) multinucleation.
These cells indicate probable infection with HPV.
16. Types of HPV
Types 6, 11Raised genital wart, mostly benign.
Types 16, 18Flat genital wart, often found in association
with invasive cervical lesion.
17 Carcinoma Vulva

1. Carcinoma vulva-risk factors


It accounts for 1 percent of all carcinomas in female and 5
percent of female genital malignancies. Predisposing factors
are (i) increased age (more than 65 years), (ii) obesity,
(iii) diabetes, (iv) vulvar dysplasia, (5) vulvar dystrophy.
2. Clinical features of carcinoma in situ of vulva
Puritus vulvae
The vulva may look healthy or there may be thick
skin or raised red areas with serpiginous margins.
3. Treatment for carcinoma in situ
Simple vulvectomy
Intermittent check-up for recurrence or other inva-
sive lesion
Topical 5 fluro-uracil produces some success.
4. Histologic subtypes of vulvar carcinoma
Squamous cells 86 % Basal cell carcinoma 2 %
Melanoma 6 % Sarcoma 2%
Adenocarcinoma 3.5 % Pagets disease 0.5 %
(Bartholins gland)
5. Risk of inguinal node involvement
Depth of invasion Inguinal node metastasis%
less than 1 mm 0
1-2 mm 8
2-3 11
3-4 26
Hence, besides tumor diameter and decreasing degree of
differentiation depth of invasion increases risk of inguinal
node metastasis.
72 Short Notes and Short Cases in Gynaecology

6. Clinical staging of vulval carcinoma


StageI The lesion is confined to vulva and is less
than 2 cms. Local lymph nodes are not
affected.
StageII The lesion is more than 2 cms but still con-
fined to vulva and inguinal lymph nodes
are not enlarged.
StageIII The lesion increases in size with involve-
ment of ipsilateral inguinal lymph node,
urethra, vagina, anus.
StageIVA All surrounding tissue including rectum,
bladder and other pelvic organs are invol-
ved. There is also meta-stases to bilateral
inguinal nodes.
IVB Distant metastasis including pelvic lymph
nodes.
7. Pagets disease of vulva
It is intraepithelial adenocarcinoma appearing as velvety red
lesion with areas of superficial white coating. An underlying
adenocarcinoma is seen in 15 to 20 percent patients. Wide
excision with 1 cm safety margin is for noninvasive lesion but
invasive lesions need radical vulvectomy plus bilateral
inguinal node resection.
8. Lymphatic drainage of vulva
The lymphatic of clitoris may drain to either groin and drain
directly to deep pelvic nodes. Superficial vulvar lymphatics
drain to superficial inguinal nodes, deep inguinal nodes and
hence to pelvic nodes.
9. Therapy of vulvar carcinoma
Large lesionsradical vulvectomy and bilateral inguinal node
resection; smaller (less than 2 cm) lesions not located in
clitoris or midlineradical hemivulvectomy and ipsilateral
node dissection.
10. Radiotherapy in vulval carcinoma
Radiotherapy is not usually practised in vulval carcinoma
due to reaction by surrounding healthy tissue. Sometimes
Carcinoma Vulva 73

deep X-ray or radium needle implantation can be applied for


5 to 7 days. Irradiation may decrease the size of growth to
make it operable. It may be applied after local excision. The
result of radiotherapy is poor in comparison to surgery.
11. Prognosis vulval carcinoma
At early stage and when there is no lymph node involvement
radical vulvectomy provides 70 to 80 percent of 5-year sur-
vival rate. Radiotherapy provides 20 to 25 percent of 5-year
survival rate and electro-coagulation 5 to 20 percent.
18 Carcinoma Cervix

1. Risk factors for carcinoma cervix


The presumed factors are as follows:
Early intercourses before the age of 17 years runs greater
risk. So early marriage poses greater risk than late marriage.
Married woman runs greater risk than virgin. Woman who
marries more than once runs still greater risk than that of
an woman with single husband. So the prostitutes run
greater risk for carcinoma cervix.
Woman form lower strata of society or from poor families
show higher incidence, may be due to early marriage and
poor personal hygeine.
Jewish, Parse and Muslim women show lower incidence
than Negros, Mexican women who show higher incidence.
HPV probably acts as a cofactor.
2. Types of carcinoma of cervix
They are of four types:
Squamous cell carcinoma
Adeno carcinoma
Mixed type (usually rare)
Undifferentiated type.
3. Modes of spread of carcinoma cervix
Direct spread by local invasion
Lymphatic spread
Blood borne spread to remote organs.
4. Groups of lymph nodes affected in carcinoma cervix
Primary lymphatic spread to nodes surrounding
cervix
Carcinoma Cervix 75

Internal iliac nodes


Obturator nodes
External iliac nodes
Sacral nodes
Common iliac nodes
Para-aortic nodes.
5. Symptoms of carcinoma cervix
Symptomless in carcinoma in situ
Irregular vaginal bleeding, post-coital bleeding
Pain in advanced stage in form of backache, low
abdominal or rectal pain
Frequency of micturition and dyspareunia
Vaginal dischargefoul smelling vaginal discharge
in advance stage when the mass is ulcerated.
6. Signs of carcinoma cervix
No visible or palpable sign in carcinoma in situ
Erosion of cervix in early stage
Ulceration of cervix, bleeds on touch
Polypoidal growth of varying size
Enlarged, hard, friable and fixed cervix in advanced
stage.
7. Methods of diagnosis of cervical cancer
Exfoliative cytology : Pap smear
Schillers test with Lugols iodine
Colposcopy
Biopsy:
Punch biopsy
Wedge biopsy
Ring biopsy
Cone biopsy.
8. The clinical staging of carcinoma cervix
Stage 0 Carcinoma in situ
Stage I Confined to cervix only
Stage I-A Pre-clinical invasive or carcinoma with
microinvasion
Stage I-B Involvement of cervical tissue
Stage II Carcinoma involving neighbouring tissue
like vagina but not pelvic wall.
76 Short Notes and Short Cases in Gynaecology

Stage II-A No parametrial involvement


Stage II-B Parametrial involvement
Stage III Carcinoma affecting pelvic wall
Stage IV Extension to neighbouring organs like rec-
tum and bladder and beyond pelvic cavity.
9. Differential diagnosis of cervical cancer
Eversion and redness around cervical OS due to in-
flammation
Hard chancre of primary syphilis
Chancroid (soft chancre)
Granuloma inguinale
Lympho granuloma venerium
Tuberculosis of cervix
Abortion of cervical pregnancy
Metastatic chorio-carcinoma
Erosion of cervix
Mucous polyp
Sarcoma of cervix
Endometriosis of cervix.
10. Treatment of carcinoma in situ of cervix
Hysterectomy if no child is required
Diathermy conisation or surgical conisation to
remove the small part of in situ lesion
Follow-up/check-up by Schillers test
No irradiation for in situ stage.
11. Treatment of stage I and II A disease
Surgery and radiation are both equally effective but in young
patients surgery is preferred because the ovaries may be pre-
served.
12. Role of radiotherapy in cervical cancer
The cervix is easily accessible for brachytherapy and is sur-
rounded by normal tissues that are radioresistant. Hence
intracavitary 10,000 rads can be delivered to tumor. The dose
of radiation falls off by inverse square of the distance from
the source.
13. Wertheims hysterectomy
It is radical hysterectomy where uterine artery is ligated at its
origin from internal iliac, uterosacral ligaments are resected
Carcinoma Cervix 77

back towards sacrum, cardinal ligaments are resected at pel-


vic side wall, pelvic lymphnodes are removed so also upper
one-third of vagina. Ovaries may be preserved in young
patients. If para-aortic nodes are positive, surgery can be
abandoned and pelvic radiation therapy with extended para-
aortic field can be given.
14. Contraindications for pelvic exenteration
Extrapelvic disease
Triad of unilateral leg edema, sciatica, ureteral obs-
truction
Tumor-related pelvic sidewall fixation
Bilateral ureteral obstruction.
15. Prognosis of cervical cancer
Stage 5-year survival
Stage I 80-85%
Stage II 50-65%
Stage III 25-35%
Stage IV 8-14%
16. Recurrence after radical surgery/radiation
After radical hysterectomy, approximately one-third of recur-
rences are in the pelvic sidewall and one-fourth in central
pelvis. Recurrence after radiation is 50 percent in parametrium
and 25 percent in cervix, uterus and upper vagina. One-year
survival in these patients is 10 to 15 percent. Pelvic exente-
ration for recurrence may be taken up if recurrence is centrally
located. Otherwise cisplatin, 5 FU and hydroxyurea may be
used (as radiosensi-tizers) along with radiotherapy.
17. Adenocarcinoma cervix
Stage for stage: There is no significant difference in survival
of patients of adenocarcinoma, vs squamous carcinoma, but
lesions initially these tend to be bulky and more poorly dif-
ferentiated. Local recurrence is more common, hence com-
bined radiotherapy and surgery are often preferred. Pap
smears are less reliable for adenocarcinoma, and the disease
is often multifocal.
78 Short Notes and Short Cases in Gynaecology

18. Advantages of irradiation in cancer cervix


All stages of invasion can be treated by radiotherapy
Morbidity and mortality is less than surgery
Medical contraindications are less than surgery.
19. Indication of surgery
Very young patients
Pelvic sepsis
Vaginal stenosis
Utero-vaginal prolapse
Associated tumors of ovary
Radiotherapy resistant cases
Recurrence after radiotherapy
Pregnancy with carcinoma.
20. Complication of Wertheims operation
Rapid matastasis to regional lymph nodes
Direct surface extension to vagina
Posterior extension to rectum
Anterior extension to bladder
Lymyhatic obstruction, pain and swelling of legs
Pain in back may increase
Pelvic infection
Hemorrhage
Formation of vaginal fistulas.
21. Management of advanced carcinoma cervix
Best possible nursing care with good nutrition
Strong analgesics to relieve pain - morphine, fenta-
nyl, etc.
Other symptomatic treatment depending on type of
complains
Surgerylike parasacral neurectomy, chordotomy,
intrathecal absolute alcohol injection at L1 veretebra
Electric coagulation to reduce small fungating and
ulcerative growth
Small external irradiation to relieve pain intermittently
Chemotherapy in selected cases for short relief.
Carcinoma Cervix 79

22. Cesium brachytherapy for cervical cancer


Stockholm technique is usually employed, 50 mg cesium tube
is placed in uterus and two ovoids containing 60 mg is packed
into vagina. The cesium is left for 22 hours. Three applica-
tions are given, second a week after the first, and third 2
weeks after the second.
19 Carcinoma
Endometrium

1. Predisposition to carcinoma of endometrium


Advancing age (55-65 years)
Hereditary tendency
Menstrual irregularity: anovulatory cycles
Late menopause, early menarche and nulliparity
Diabetes mellitus
Estrogenic overactivity, obesity, estrogen replacement
Simple endometriosis
Atypical endometrial adenomatous hyperplasia.
2. Presentation of endometrial carcinoma
Irregular vaginal bleeding; Postmenopausal bleeding
Vaginal discharge
Dull abdominal pain
Bloody menopause.
3. Staging of endometrial cancer
IA Tumor limited to endometrium
IB Invasion less than myometrium
IC Invasion more than myometrium
II A Endocervical glandular involvement only
II B Cervical stroma invasion
III A Invasion of adnexa/serosa
III B Vaginal metastasis
III C Metastasis to pelvic/para-aortic nodes
IV A Invasion of bladder/bowel mucosa
IV B Distant metastasis, inguinal nodes.
4. Pap smear in endometrial cancer
Only 30 to 50 percent of patients with endometrial cancer
have abnormal pap smear and if pap smear is abnormal the
Carcinoma Endometrium 81

cancer is already in advanced stage. If normal endometrial


cells are seen in pap smear from a postmenopausal woman,
endometrial biopsy be performed since 6 percent of these
women have endometrial cancer. Upto 43 percent of patients
with preoperative diagnosis of atypical endometrial adeno-
matous hyperplasia in D and C have endometrial cancer on
hysterectomy.
5. Prognostic factors in endometrial cancer
Degree of differentiation
Histologic type
Depth of myometrial invasion
Cervical stromal involvement
High grade tumors or that deeply invade myometrium or
involve cervix are more likely to have invaded adnexa and
lymph nodes, placing patient at advanced stage.
6. Prognosis in endometrial cancer
Approximately 75 percent of endometrial cancer are in grade
I at diagnosis. In stage I, 5-year survival is 70 to 80 percent. In
advanced disease irradiation provides 20 to 40 percent
5-year survival rate.
7. Role of hormone therapy in endometrial cancer
Many low grade endometrial cancers have estrogen and
progesterone receptors and respond to progesterone therapy
but overall response rate is less than 30 percent. Thus, it is
often used for metastatic disease.
8. Uterine papillary serous adenocarcinoma
It is a variant of endometrial cancer whose histology resembles
ovarian serous carcinoma. It tends to spread intraperitoneally
like ovarian cancer and has higher incidence of nodal and
distant metastasis.
9. Relation of tamoxifen to endometrial cancer
Tamoxifen therapy induces or promotes endometrial adeno-
carcinoma but the risk is dose and time-dependent. The risk
is 2 to 3 times that of general population.
20 Gestational
Trophoblastic Disease

1. Molar pregnancy
The two types of molar pregnancy are complete and partial.
A complete hydatidiform mole arises from the fertilization of
an empty ovum, i.e. the nucleus is absent. The sperm dupli-
cates its own chromosomes. The most common chromosome
pattern is 46XX, all paternally derived. No embryonic tissue
is present. A partial mole is rare, its chromosomal pattern is
triploid, again paternally derived. The fetus usually survives
for 8 to 9 weeks. Malignant sequelae occur in 5 to 10 percent
with partial mole and 20 percent with complete mole.
2. Histologic appearance of hydatidiform mole
The tissue consists of diffusely hydropic chorionic villi and
hyperplastic trophoblasts. The cystic villi give appearance
of cluster of grapes.
3. Presentation of complete mole
Vaginal bleeding
Uterine size greater than period of amenorrhoea
Hyperemesis gravidarum
Pre-eclampsia
Hyperthyroidism
Trophoblastic pulmonary emboli.
4. Medical complications of molar pregnancy
Hyperthyroidism
Hypertension
Anemia
Hyperemesis
Bilateral theca
Lutein cysts.
Gestational Trophoblastic Disease 83

5. Diagnosis of hydatidiform mole


Snow storm pattern in ultrasound or soap bubble
appearance
hCG more than 1000000 units/L.
6. Treatment of molar pregnancy
Dilatation and suction with curettage
Hysterectomy if patient does not wish to retain fertility.
7. Post-evacuation follow-up
Monthly hCG estimation for first six months and then
every 2 months for another six months
Avoidance of pregnancy for 1 year (not by OCP but
by barrier method).
A rise in hCG indicates incomplete removal or onset of
choriocarcinoma. Almost all malignant sequelae occur within
six months of evacuation. Patient with prior mole have 10 fold
increased risk of mole in next pregnancy.
8. Gestational trophoblastic disease (GTD) genesis
GTD may develop after any gestation, including term and
ectopic pregnancy or spontaneous abortion but the majority
occur after hydatoidi from mole (40-50%).
9. Morbid anatomy of choriocarcinoma
It is solid plum-colored or purple, friable, dark, hemorrhagic
nodules appearing in uterine cavity. It always infiltrates and
causes bleeding and discharge.
10. Histological picture of choriocarcinoma
It shows irregular growth of cytotrophoblasts and sincytio-
trophoblast cells. The cells are actively growing, many show
typical mitotic division. There is no villus pattern.
11. Symptoms and signs of choriocarcinoma
Symptoms
History of abortion or hydatidiform mole
Intermittent severe vaginal bleeding
Vaginal discharge
84 Short Notes and Short Cases in Gynaecology

Associated symptoms of involvement of remote


organs like lungs, in advanced cases.
Signs
Sub-involution of uterus
Secondaries in vulva and vagina
Enlarged, bilateral, polycystic ovaries.
12. Types of GTD
a. Invasive mole
b. Choriocarcinoma
c. Placental site tumor.
13. Staging of GTD
Stage I Confined to uterine body
Stage II Metastasis to pelvis or vagina
Stage III Metastasis to lungs
Stage IV Distant metastasis.
14. Treatment of GTD
Methotrexate 40 to 50 mg/m2/week
Dactinomycin 1.25 mg/m2 IV every 2 weeks
Treatment be continued till hCG becomes normal and then 2
more cycles are given.
Patients with one or more high risk factors need combina-
tion chemotherapyalternating etoposide methotrexate, or
dactinomycin with cyclophosphamide and vincristine.
Cisplatin monotherapy is also very effective.
15. Choreoadenoma destruens (invasive mole)
Here the moles penetrate uterine wall causing uterine enlarge-
ment and bleeding. They may penetrate to parametrium to
cause intraperitoneal bleed. The moles still remain benign.
16. Placental site tumor
Though extremely rare it occurs after nonmolar gestation.
Tumor growth is very slow but when metastasis occurs it is
usually fatal.
17. Prognosis in choriocarcinoma
Stage I, II and III disease100 percent survival with chemo-
therapy
Stage IV70 percent survival with chemotherapy.
21 Tumors of Ovary

The incidence of ovarian tumor increases with age. Nulliparous


women have increased risk and OCP provide some protec-
tion. Those with breast cancer have two fold risk less than
5 percent have familial tendency. CA-125 is elevated in 80
percent cases with epithelial cancers and 75 percent present
with stage III disease.
1. Types of ovarian tumors
Innocent ovarian tumors
Malignant ovarian tumors
Metastatic ovarian tumors
Tumours arising from connective tissue of ovary
Tumours arising from ovum
Tumours arising from primitive mesenchyme.
2. Innocent or benign tumors
Simple serous cyst adenoma
Papillary serous cyst adenoma
Pseudomucinous cyst adenoma
Breners tumour.
3. Malignant tumors of ovary
Serous cyst adeno-carcinoma
Pseudomucinous cystadeno carcinoma
Solid carcionoma
Meso nephroma.
4. Metastatic tumors of ovary
Typicalfrom primary carcinoma of stomach, intes-
tine breast, and uterus
AtypicalKrukenberg tumours.
86 Short Notes and Short Cases in Gynaecology

5. Tumors arising from connective tissue of ovary


BenignFibroma
Malignantsarcoma.
6. Tumors arising from the ovum
Dermoid cysts
Solid teratoma
Chorion epithelioma
Struma ovaries.
7. Tumors arising from primitive mesenchyma
I. Feminising (female hormone producing)
Granulosa cell tumors
Thecacell tumors
Luteoma.
II. Neuter (Neutral) Disgerminoma (Seminoma)
III. Virilising(Male hormone producing)
Arrhenoblastoma
Supra renal cortical tumors
Hilus cell tumor
IV. MixedGynandroblastoma.
8. Most common and largest encountered ovarian tumors
Pseudo Mucinous cyst adenoma. It comprises 70 percent of
all ovarian tumors.
9. Symptoms of pseydomucinous cyst adenoma
Enlargement of adbomen and feeling of a mass.
Very rarely menstrual disorders
Pressure symptoms like:
Respiration embarrassment
Bilateral oedema feet
Frequency of micturition
Rarely gastrointe stinal symptoms
Occasionally, accidental torsion of cyst causing acute
abdomen with severe pain, shock or collapse.
10. Signs of pseudomucinous cystadenoma
Huge abdominal swelling
The abdominal wall can be moved over the swelling
Tumors of Ovary 87

On palpation the upper and lateral wall of tumor can


be defined
The surface of tumor is smooth, the consistency
being tense, cystic
Some times fluid thrill can be detected
On asculation of tumour over abdomen it is silent
On percussiondull over centre and resonant over
flanks
The lower pole of tumor can be palpated through
anterior or posterior fornix.
11. Differential diagnosis of ovarian cyst
Full bladder
Pregnant uterus
Fibroid of uterus
Tubo ovarian mass
Ascites
Other abdominal tumours like hydronephrosis,
mesenteric cysts and tuberculous peritonitis.
12. Clinical diagnosis of large ovarian cyst
The diagnosis can be made from the following cardinal signs:
By mannual P/V examination when the ovarian tumours
are raised by abdominal hand, the cervix remains station-
ary to vaginal finger unlike fibroid of uterus where the
cervix is also raised.
In large ovarian cyst associated with ascites, the percus-
sion note over tumour or centre of abdomen is dull and
flanks are resonant, whereas in ascites, it is vice versa.
13. Investigations in doubtful diagnosis
Thorough examination under anaesthesia
Ultrasound of pelvis and obdomen
Culdoscopy or peritoneoscopy
Hystero-salpingography may show displacement of
tube by tumor
Radiography may show soft tissue shadow or teeth
for dermoid cyst
IVP to exclude renal swellings
Barium meal X-ray to exclude gestrointestinal tumors
88 Short Notes and Short Cases in Gynaecology

Exfoliative cytologyMicroscopic exam of peritoneal


fluid may show presence of malignant cells.
CA-125.
14. Characteristics of Breners tumor
It is slow growing benign solid tumor with wide varia-
tion in size
It is small round, grey in colour and has consistency
like that of a myoma of uterus
It is rarely seen and the incidence is 1.7 percent of all
ovarian tumors
It is usually unilateral
It occurs at menopause or during post menopausal
period
Microscopically it contains polyhedral epithelial cells
surrounded by fibrous connective tissue and filled
with amorphous colloidal material.
15. Clinical features of malignant ovarian tumors
Age incidenceMost of the malignant tumours arise
during menopause or postmenopausal period
PainMalignant ovarian tumours invariably cause
abdominal pain than benign ones
BleedingIn women of postmenopausal age vaginal
bleeding is a typical feature of granulosa cell tumors
Bilateral SwellingsInvariably in all cases bilateral
tumors are seen
AscitesDevelopment of fluid in peritoneal cavity
is an important sign of malignant ovarian tumors
Fixation to neighboring tissue is almost present
Severe cachexiaExtreme loss of body weight and
edema of limbs
X Ray shows bony metastasis or pulmonary deposits.
16. Complications of ovarian tumors (benign and malignant)
Torsion
Rupture of cyst
Hemorrhage
Degeneration
Inflamation
Tumors of Ovary 89

Intestinal obstruction
Impaction.
17. Clinical stagings of malignant ovarian tumors
Stage-I Tumour confined to ovaries only
Stage-Ia Growth limited to one ovary, without
ascites
Stage-Ib Growth limited to both ovaries without
ascites
Stage-Ic Growth limited to both ovaries with ascites
Stage-II Growth involving one or both ovaries, with
pelvic infiltration
Stage-IIa Extension to uterus or tubes or other ovary
Stage-IIb Extension to other pelvic organs
Stage-III Growth involving one or both ovaries with
intraperitoneal metastasis inside pelvis
Stage-IV Direct metastasis.
18. Krukenbergs tumor
This is a metastatic malignant ovarian tumor, the primary
source being gastrointestinal malignancy.
The tumor is almost bilateral
They have smooth surface and freely moves in pelvis
with well developed capsule
They are firm, solid growths of moderate size
The ovary usually maintains its shape. Sometimes become
kidney shaped or lobulated
There is no tendency to form adhesion with neighboring
viscera
The cut surface shows degenerated, hemorrhagic and
cystic areas.
The tumor contains cellular or myxo matous stroma
Large signet-ring cells are seen
Clusters of epithelial cells arranged in acini show mucoid
epithelial changes.
19. Granulosa-cell-tumor
It contitutes 10 percent of all solid malignant ovarian
tumors
It can occur at any age. The clinical features depend
on the oestrogenic activity
It is usually unilateral (95%)
90 Short Notes and Short Cases in Gynaecology

When occurring before puberty precocious puberty


occurs with early development of secondary sex char-
acters
When occurs in adult life amenorrhoea and excess
blood estrogen level is found
When occurs after menopause there is postmeno-
pausal bleeding with some hypertrophy of breasts.
20. Granulosa-cell-tumor malignant
It is not definite to grade it as malignant from time of detection.
The degree of malignancy varies. The following features if
found may warn as malignant.
Large size
Rapid growth with necrosis and hemorrhage
Adhesion to neighboring organs
Formation of ascites.
21. Disgerminoma
It is a neutral mesenchymal tumor of ovary corresponding to
seminoma of testis in male.
It is commonly seen in female pseudohormophrodites
It occurs at any age commonly between 12 to 20 years
It does not cause sexual abnormality.
It is usually small but may attain big size to cover whole
of abdomen.
It may become cystic due to necrosis, degeneration and
hemorrhage
It is greyish, lobulated with well defined thin fibroid cap-
sules.
Radiotherapy in small tumor as the tumor is radiosensitive
Surgical removal in bigger tumor with signs of malignancy.
22. Arrhen oblastoma
It is a rare tumor and constitutes less than 1 percent
of ovarian tumors
It is common in age group of 20 to 30 years
It arises from certain male directed cell present in
reteovarii
It is a slow growing tumor and is usually unilateral
It is small solid tumor and remains inside ovary.
Tumors of Ovary 91

Clinical features include


Atrophy of breasts, Amenorrhoea, Hirsutism, Hyper-
trophy of clitoris, Excess acne, Deepening of voice
On pelvic examination small to moderate size ovarian
swelling can be detected varying from 2 to 3 inches in
diameter.
23. Prognosis in ovarian cancer
Five-year survival in patients with stage I disease is 80
percent. Grade I stage I tumors have 5-year survival of 95
percent but 5-year survival in stage III or IV disease is only
25 to 30 percent.
24. Surgical staging
Surgical staging in the absence of obvious stage III disease
includes (i) peritoneal washing, (ii) multiple peritoneal biop-
sies, (iii) pelvic and para-aortic node sampling, (iv) infracolic
omentectomy.
25. Postoperative chemotherapy
Cisplatin 50 to 100 mg/m2 often combined with palcitaxel/
cyclophosphamide for 5 cycles improves survival. Survival
is better in patients with smaller residual disease than in
patients with larger residual disease. Debulking to reduce
residual tumor size to less than 2 cm is optimal for better
results with chemotherapy.
Cisplatin intraperitoneal therapy is used in patients with
residual disease after second surgery. Patients with minimal
residual disease who are chemoresponsive may benefit from
high dose chemotherapy with bone marrow transplant.
Tumors of low malignant potential are epithelial tumors
with excellent prognosis. Even if they have spread to abdo-
men, 5-year survival is above 80 percent.
26. Histogenesis of germ cell tumors
Germ cell Dysgerminoma

Embryonal carcinoma

Embryonic tumors Extra embryonic

Teratoma Polyembryonic Endodermal Choriocarcinoma


sinus tumor
92 Short Notes and Short Cases in Gynaecology

27. Tumor markers for ovarian germ cell tumors


hCG AFP LDH CA 125
Mixed germ cell tumor + + + +
Embryonal carcinoma + + +
Endodermal sinus tumor +
Dysgerminoma + +
Immature teratoma
Choriocarcinoma +
28. Teratomas of ovary
a. Immature
b. Mature
i. Solid
ii. Cystic
Dermoid cyst (mature cystic teratoma)
Dermoid cyst with malignant transformation
c. Monodermal or highly specialised
i. Struma ovarii
ii. Carcinoid
iii. Struma ovarii plus carcinoid
29. Germ cell tumors
Germ cell tumors constitute 20 percent of all benign and
malignant ovarian neoplasms but only 3 to 5 percent of them
are malignant. They are more frequent in children and adoles-
cents comprising 60 to 70 percent of ovarian neoplasms in
female age below 20 years. One-third of germ cell tumors in
patients of this age group are malignant.
22 Infertility

Failure to conceive after 1 year of unprotected intercourse is


called infertility. It affects 10 to 15 percent of couples in repro-
ductive age (15-45 year old).
1. General causes of infertility
Male factor (30-40%)
Ovulatory dysfunction10 to 25 percent
Pelvic factor (tubal disease, endometriosis)30 to 50
percent
Cervical factors5 to 10 percent
No demonstrable cause in 20 percent.
2. Normal semen analysis
Volume more than 2 ml
pH 7.2 to 7.8
Sperm count more than 20 million/ml
Sperm motility more than 50 percent actively notice
Spermorphology more than 50 percent normal.
3. Work-up in female infertility
US pelvis (for uterus and ovaries)
Dilatation and curretagefor endometrial status
Hysterosalpingographyfor uterine and tubal
anomalies
Laparoscopy for endometriosis
Postcoital test
Antisperm antibody.
4. Postcoital test
Within 12 hours of intercourse and at midcycle, a womans
cervical mucus is assessed for quality, quantity and number
of motile sperms. A normal postcoital test demonstrates
abundant clearly, relatively acellular mucus with more than
5 motile sperm per HPF.
94 Short Notes and Short Cases in Gynaecology

5. Tubal blocktreatment
Distal tubal blockSalpingostomy
Proximal tubal blockHysteroscopic tubal balloon
dilatation
Both proximal and distal tubal diseaseIVF.
6. Symptoms of PCOs
Infertility in 74 percent
Hirsutism in 69 percent
Amenorrhoea in 51 percent
Obesity in 41 percent.
7. Abnormal laboratory values in PCOS
Raised testosterone, DHEA, androstenodione and LH
Low level of FSH and estradiol, SHBG
LH/FSH ratio of 3:1 (normalis 1.5:1)
Raised estrone due to peripheral conversion from
androstenodione.
8. Symptoms of elevated testosterone
Hirsutism
Insulin resistance
Alanthosis nigricans (gray brown velvety area of hy-
perpigmented skin in axilla, back of neck).
9. Treatment of hirsutism of PCOS
Spironolactone
GnRH analog
Low-dose OCP (not containing androgenic norgestril).
10. Induction of ovulation in PCOS
Clomiphene; 90 percent will ovulate and 50 percent
will become pregnant
If clomiphene unsuccessful -FSH or menopausal
gonadotropin
If both fail wedge resection or laser is destruction of
ovarian stoma (only short-term benefit, adhesion limit
is long term benefit).
Metformin and
11. Inheritence pattern of PCOS
Sex-linked dominant.
23 Anovulation,
Polycystic Ovary

Anovulatory women may complain of amenorrhoea, irregular


menses or heavy menstrual flow. Molimina, the feeling of
impending menstruation and dysmenorrhea are typically
absent in anovulatory cycles. Since there is no corpus
luteum formation and progesterone secretion, endometrial
shedding is disorganized and menstrual bleeding is unpre-
dictable in frequency and quantity.
Anovulation is due to inhibition of pulsatile release of
GnRH from hypothalamus due to stress, strenuous exercise,
weight loss, or Kallman syndrome (defective GnRH secretion
with anosmia). Often the ovaries fail to respond to FSH and
LH as in gonadal dysgenesis, resistant ovary syndromes or
premature ovarian failure. Often the pituitary is at fault due to
tumor (prolactinoma) or cranyopharyngioma.
1. Approaches to pituitary anovulation
Estimate serum prolactin, TSH, FSH
If prolactin is high or TSH is high treat with
bromocriptine or thyroid replacement
When prolactin, TSH and FSH, LH are normalgive
clomi-phen citrate, a nonsteroidal antiestrogen that
prevents negative feedback of estrogen on pituitary.
Start 50 mg daily on days 5 to 9 of cycle and increase
to 200 to 250 mg daily on cycle day 5 to 9 till ovulation
returns.
2. Diagnosis of ovulation
in basal body temperature 0.4 to 0.6F
Serum progesterone > 3 ng/ml
Collapse of dominant follicle in US.
96 Short Notes and Short Cases in Gynaecology

3. Failure of clomiphene therapy


If ovulation fails to occur after follicular growth with clomi-
phenegive hCG 10,000 IU once follicle diameter is 20 mm.
This triggers LH surge and induces ovulation. Although 80
percent of women ovulate with clomiphene only 40 percent
conceive. Clomiphene is unlikely to be effective in hypo-
estrogenic women. Hence FSH is to be administered early in
follicular phase followed by hCG.
4. Other options of ovulation induction
GnRH may be given to stimulate pituitary FSH, LH secretion.
However, it is to be administered in pulsatile manner by
infusion pump. The chances of multiple birth and ovarian
hyperstimulation syndrome is lower.
5. Inherent risks of ovulation induction
The risks include multiple gestation and ovarian hyperstimu-
lation syndrome. The latter is potentially life-threatening
characterized by ovarian enlargement, ascites, hemoconcen-
tration and hypercoagulability.
6. Criteria for diagnosis of polycystic ovary syndrome
(PCOS)
Perimenarchial onset of menstrual irregularity
Obesity
Physical and laboratory evidence of androgen excess
Chronic anovulation
Inappropriate gonadotropin secretion.
7. Syndromes confused with PCOs
Hyperprolactinemia
Late onset adrenal hyperplasia
Ovarian adrenal neoplasm
Cushings syndrome.
24 IVF-ET

1. Major steps in IVF-ET cycle


Controlled ovarian hyperstimulation
Transvaginal oocyte retrieval
Fertilization of oocyte
Embryo culture until cleavage (4-8 cell)
Transcervical transfer of embryo into uterine cavity.
2. Major indications of IVF-ET
Tubal disease
Immunologic factors
Male factor infertility
Ovulatory dysfunction
Genetic disorders
Idiopathic infertility
Endometrial unresponsiveness to medical/surgical
therapy.
3. Factors adversely affecting outcome in tubal surgery
Presence of large hydrosalpinx (more than 3 cm)
Absence of discernible fimbria
Severe pelvic adhesive disease
Salpingostomy after distal tubal block
Less than 4 cm of tube remaining after tubal ligation
Repeated ectopic pregnancy
Endometriosis with tubal or ovarian involvement.
In all above cases chances of successful viable delivery
is only 20 percent.
4. Reasons for ovulation stimulation in IVF-ET
This is because 3 to 4 oocytes are to be recovered and im-
plantation of same number of embryos is the rule. Successful
98 Short Notes and Short Cases in Gynaecology

pregnancy rates after embryotransfer are directly related to


the number of embryos transferred. The excessive recovered
embryos can be cryopreserved for subsequent ET.
5. Donated oocytes
Women with menopause secondary to premature ovarian fail-
ure, gonadal dysgenesis, resistant ovary syndrome benefit
from IVF with donated oocytes. Women who carry defective
genes can become mothers with donated oocytes.
6. Ovulation inducing agents
Clomiphene citrate (antiestrogen)
Human menopausal gonadotropin (50:50 FSH and LH)
GnRH analogs
Human chorionic gonadotropin (mainly LH).
7. Protocol for controlled ovarian stimulation
Down regulation with GnRH analog followed by gonadotropin
stimulation. Its advantages are (i) greater number of synchro-
nous oocytes, (ii) avoidance of premature LH surge, (iii) lower
incidence of premature luteinization.
8. Monitoring of HMG therapy in IVE-ET
Number and size of lead follicle
Serum estrogen level and rate of rise
Length of ovarian stimulation
Status of vaginal cytology and cervical mucus char-
acteristics
Evidence of increase in LH or progesterone level.
9. Role of estradiol assay in IVF-ET
Baseline estradiol level of more than 100 pg/ml means active
follicular development and is against ovulation induction
therapy. Low estradiol level on 2 or more days suggests that
HMG dosage is inadequate. A peak estradiol level less than
500 pg/ml is associated with poor outcome. The serum estro-
gen pattern associated with highest pregnancy rate include
(i) progressive rise in estrogen level throughout HMG
administration, (ii) continued rise between last day of HMG
dose and HCG dose, (iii) further rise of estrogen level after
IVF-ET 99

HCG dose. Estradiol level more than 3000 pg/ml is associated


with ovarian hyperstimulation syndrome.
10. Why embryos are transferred in IVF-ET
Transfer of less than 4 embryos is associated with lower preg-
nancy rate
Number of embryo transferred Pregnancy rate
1 9-10
2 12-15
3 15-20
4 20-25
25 Menopause

Menopause is defined as cessation of menses for a minimum


of 6 months because of inadequate ovarian follicular deve-
lopment and waning of estrogen production. The climacteric
is an extended period of gradually declining ovarian function
often beginning years before and lasting years after meno-
pause itself. Menopause occurs between 45 to 55 years.
Natural menopause occuring before 40 years is termed
premature ovarian failure.
1. Symptoms of menopause
Hot flashes Dyspareunia
Irritability, fatigue Vulvar pruritus
Depression, headache, Urinary frequency,
insomnia dysuria
2. Lab diagnosis of menopause
FSH > 40 mIU/ml persistently.
3. Osteoporosis and menopause
Spinal compression, and hip fractures are common. Factors
known to increase the risk of osteoporosis are reduced weight
for height, early menopause, smoking, sedentary life, steroid
intake, high caffeine intake, low vitamin D and calcium.
4. Contraindications to ERT
Active thromboembolism
Chronic liver disease
Abnormal uterine bleeding
Migraine, seizure
Endometrial cancer and atypical adenomatous hyper-
plasia.
Menopause 101

5. Phytoestrogens
They are selective receptor modulators found in legumes
(soy). Which mostly bind to estrogen receptor , hence acting
as estrogen on bone, brain and vasculature but no stimula-
tory effect on breast and uterus. 60 g of isodated soy protein
daily reduces hot flushes in 45 percent over 12 weeks.
6. Cardiovascular effects of ERT
HDL LDL atherosclerosis.
7. Diagnosis of osteoporosis
X-ray spineprominent vertical striations, (only when
> 50 percent calcium is lost)
Bone densitometry (dual photon, quantitative CT)
US densitometry.
8. Control of postmenopausal symptoms
Estrogen replacement therapy (ERT)
Clonidine for control of hot flushes
Medroxy progesterone for hot flushes
Tamoxifen provides some bone protection
Raloxifene, a selective estrogen receptor modulator
60 g daily to protect against bone loss in women who
can not take estrogen (breast/endometrial cancer).
9. Advantages of ERT
Prevention of osteoporosis
Restoration of menses when patient wants this for
psychological reasons
Better control of lot flushes, mood changes, vaginal
dryness, urinary incontinence, mood changes, breast
atrophy, and skin wrinkling
Improved serum lipid profile with reduction in CAD
risk.
10. Side effects of ERT
Weight gain, increased appetite, breast tenderness,
edema.
4 to 8 fold increased risk of endometrial adenocarci-
noma
Atypical adenomatous hyperplasia of endometrium
Breast cancer risk is not appreciably increased.
102 Short Notes and Short Cases in Gynaecology

11. Role of Tibolone


It is a 19-norsteroid with estrogenic and progesteronal
properties and weak androgenic activity. Daily dose of 2.5
mg controls postmenopausal symptoms and improves libido
osteoporosis while decreasing CVS risk. It should be avoided
in malignancy of uterus, breast, epilepsy, migraine and throm-
boembolism.
Short Cases
Case 1

A young lady married since two years and having normal


sexual relationship without contraception has not conceived
so far.
1. What do you think of this lady?
Primary infertility.
2. What are the terms, infertility and sterility?
Infertility is usually defined as the failure to conceive
after one year of intercourse without contraception.
Sterility implies an intrinsic inability to achieve preg-
nancy. So, infertility is synonymous with substerility.
3. Chances of pregnancy in normal young couples.
Eighty-five percent of couples with unprotected inter-
course achieve pregnancy within one year and 90
percent within 2 years. The chances of pregnancy is
20 to 30 percent per month.
4. Age related fertility in women.
The age range of maximal fertility in women is 20 to 24
years. At this age only 15 percent remain infertile after
one year; this rate increasing to 37 percent for those of
30 to 35 and 48 percent for women between 35 to 39.
Major factors involved in infertility and their approxi-
mate frequencies.
Male factors 40 percent cases
Cervical factor 5 to 10 percent
Uterine factor 5 to 10 percent
Ovulatory factor 15 to 25 percent
Tubal factor and endometriosis 30-50 percent
15 to 20 percent of couples diagnosed as infertile,
conceive without treatment, while appropriate therapy
(not IVF) will achieve pregnancy in 50 to 60 percent.
106 Short Notes and Short Cases in Gynaecology

Twenty percent of couples remain infertile in spite of


treatment and in 10 percent, no cause for infertility can
be identified.
5. Male factors for infertility
Endocrine factors
Hypothalamic dysfunction (Kallmanns syndrome)
Pituitary failure (tumor, radiation, surgery)
Hyperprolactinemia (tumor, drugs)
Thyroid and adrenal disease
Anatomical
High hypospadias
Congenital abnormality of ejaculatory system
Abnormal spermatogenesis
Chromosomal anomalies (Klinefelter)
Mumps orchitis
Cryptorchidism
Radiation exposure
Varicocele
Abnormal motility
Absent cilia
Sperm antibody
Sexual dysfunction
Retrograde ejaculation
Impotency
Normal semen parameters
Liquefaction 30 minutes
Count 20-250 million/ml
Volume 2-5 ml
Motility more than 50 percent
Morphology more than _60 percent normal
pH 7.27.8
No puscells
6. Normal duration of spermatogenesis cycle
Seventy to Ninety days. Hence effect of any drug given
to improve spermatogenesis can only be seen after
3 months and similarly, exposure to drug, viral illness
only is reflected in sperm count after similar interval.
7. What is postcoital test?
Post coital test is done to assess sperm-cervical mucus
interaction in the preovulatory period when the estrogen
Case 1 107

level is high. The mucus is thin, watery and a cellular


and its dries with a fern pattern. Such mucus collected
several hours after intercourse is seen under microscope
for number of viable and motile sperms. Normally 1000
sperms should reach oocyte for penetration and ovula-
tion.
8. What to do, when mucus is normal but sperms are
immobile?
This means that the semen contains sperm autoanti-
bodies or the wife has circulating antibodies against
sperms (head, tail or midpiece).
9. When all previous tests are negative What is to be
done?
A sperm penetration assay test to know ability of sperm
to penetrate the zona free hamster egg or in vitro fertili-
zation.
10. Predisposing factors for sperm antibody production
Testicular trauma, surgery on testis/vas, orchitis.
11. Tests for ovulation
1. Serum progesterone in third week of cycle if exceeds
5 mg/ml, ovulation has occurred.
2. Urine LH surge after ovulation.
3. Intravaginal US to assess and follow preovulatory
follicle and its rupture.
4. Rise in basal body temperature in luteal phase.
5. Secretory endometricem in endometrical biopsy in
luteal phase.
6. Premenstrual molimina.
7. Cervical mucus changes in luteal phase-i.e., the
mucus becomes thick, tachy, cellular with loss of fern
pattern.
12. What to do nextif ovulation is positive?
1. Hysterosalpingogram and hysteroscopy to exclude
malformation of uterus, submucus leomyoma, intrau-
terine synechiae, intrauterine polyps, salpingitis
isthmica nodosa, tubal occlu-sionproximal or dis-
tal. Hydrotubation with ultrasound examination is
rather better than conray hysterosalpingogram.
108 Short Notes and Short Cases in Gynaecology

2. Laparoscopy to see fimbrial adhesion, peritubal cysts,


endometriosis, ovarian disease.
13. How to differentiate septate uterus from bicornuate
uterus?
In hysteroscopy of HSG, both have similar appearance.
In laparoscopy, the fundal surface of septate uterus looks
normal, but a cleft is seen in bicornuate uterus.
14. When to suspect possibility of tubal factors?
History of previous tubal pregnancy, previous tubal
surgery, ruptured appendix, IUD use, septic abortion,
chlamydia/gonorrhoeal infection. Despite these histori-
cal clues, 50 percent of those with tubal damage have
no history of above factor.
Best surgically treatable cause of male infertility vari-
cocele.
15. What is the role of testicular biopsy in infertility?
Testicular biopsy is done to assess spermatogenesis. If
spermatogenesis is normal but still patient is azoo-
spermic, a block in the Vas or ejaculatory system is like.
Ultrasound can faithfully image the seminal vesicles and
any block in their ducts.
16. What is success rate and pregnancy rate after vaso-
vasostomy?
In experienced centers, 75 to 90 percent patency rates
with 33 to 70 percent pregnancy rates can be achieved
after vaso vasostomy. The successful outcome depends
upon quality of sperm, which is influenced by length of
time since occlusion and presence or absence of sperm
auto antibodies.
17. What to do when sperm antibody is positive?
When sperm antibody is positive, in vitro fertilization
should be advised and the wife should receive a course
of period prior to receiving the embryo, with a success
rate of 30 to 50 percent.
18. Role of varicocelectomy in the treatment of male
infertility.
A varicocele can be demonstrated in 15 percent of men,
but the incidence of infertility in varicocele patients is
Case 1 109

5 percent. Abnormal sperm counts, motility and mor-


phology are often associated with varicocele and 2/3 of
these patients show improved sperm parameters after
surgical correction but with a postoperative pregnancy
rate of only 25 to 50 percent.
19. What to do, when semen is normal, but postcoital test
is poor?
Intrauterine insemination of washed concentrated
sperms; placement of sperm directly into tubes around
time of ovulation or intraperitoneal insemination by
culdocentesis.
20. What is the hope foroligospermic males not respond-
ing to clomiphene- FSH?
In vitro fertilization
Artificial insemination
21. How to treat non-ovulating females without prolacti-
noma?
Clomiphene trial
hMg (human menopausal gonadotropin) followed
by hCG (human chorionic gonadotropin).
Buseriline or nafariline (gonadotropin releasing hor-
mone analog) trial, when hypothalamic deficiency is
suspected.
85 to 95 percent success rate in inducing ovula-
tion with hMG-hCG and buseriline.
22. What should be done if the lady has high prolactin
level?
CT scan and NMR to document prolactinoma.
Exclusion of hypothyroidism by T3, T4, TSH study
(hypothyroidism increases prolactin).
Detailed drug history as many drugs increase pro-
lactin level.
Bromocriptine therapy to normalize prolactin and
once pregnancy has occurred, the drug is to be dis-
continued.
23. What are the common pelvic factors for female infer-
tility?
Endometriosis
Salpingitis
110 Short Notes and Short Cases in Gynaecology

24. What is the role of fibroids in infertility?


The role of fibroid in infertility is unclear. Surgery is
reserved for submucous myoma or myoma distorting
uterine cavity.
25. How cervical factors account for infertility?
When the cervix is altered by congenital malformation
or by postsurgical treatment that has rendered endocer-
vical glands absent or nonfunctional, the altered or
absent mucus accounts for poorsperm movement and
infertility.
26. Scope of endoscopic surgery in infertility.
Adhesiolysis in endometriosis
Tubal surgery (fimbriolysis)
Wedge resection of ovary.
27. Major indications for in vitro fertilization.
Tubal disease
Endometriosis unresponsive to treatment
Male factor infertility
Immunologic factors
Idiopathic infertility
28. In patients undergoing tubal surgery, which factors
are associated with lower subsequent pregnancy rates?
Presence of large hydrosalpinx (>_ 3 cm)
Absence of discernible fimbria
Severe pelvic adhesive disease
Distal tubal obstruction
Less than 4 cm of healthy tube left after surgery.
Repeated tubal pregnancy.
29. Advantage and disadvantage of clomiphene.
Less expensive
Low risk of ovarian hyperstimulation
High spontaneous abortion rate
Because of antiestrogen effect, adversely affects en-
dometrial maturation and cevical mucus.
30. Advantage of hMG in ovulation induction.
Dependable ovarian stimulation
No antiestrogen effect
Production of multiple preovulatory follicles.
Case 1 111

hMG contains both FSH and LH and as 3 to 4


oocytes are required for IVF, hMG is preferred.
31. Why 4 embryos are transferred for IVF?
Transfer of fewer than 4 embryos has been associated
with lower pregnancy rates.
No. of embryos transferred Pregnancy rate percent
1 9-10
2 12-15
3 15-20
4 20-25

32. What are the in vitro fertilisation techniques avail-


able?
IVF Gamete intrafallopian transfer
Zygote intrafallopian transfer
Intraperitoneal transfer
Uterine transfer
33. Over all success rate of IVF.
25 to 30 percent
Case 2

A 50-year-old female is complaining of low back pain and hot


flushes. She has amenorrhoea for past one year.
1. What do you think of this lady?
A case of postmenopausal syndrome.
2. Which symptoms are frequently associated with meno-
pause?
Osteoporosis
Hot flushed
Irritability
Insomnia
Depression
Dyspareunia
Vulvar pruritus
Urinary frequency and dysuria
3. Why menopause occurs?
1. Oocytes responsive to gonadotropins disappear from
ovary.
2. Remaining oocytes in ovary do not respond to gona-
dotropin.
4. Hormonal changes associated with menopause?
Loss of ovarian and rostenodione, DHEA
Decreased estradiol
Raised FSH and LH
High GnRH.
5. Effects of estrogen deficiency on various organ sys-
tems.
Atrophy of bladder mucosa and vaginal epithelium,
loss of urethral tone accounting for cystitis, dyspare-
unia and frequency
Case 2 113

Osteoporosis particularly of spine and hip probably


due to reduced calcitonin secretion.
High cholesterol, LDH and triglycerides and thus
increased propensity for IHD.
6. What are the causes of hyperestrogenemia in post-
menopausal women?
Increased estrogen from granulose cell tumors
Increased production of precursor androgens as in
Brenners tumors, dysgerminoma, Krukenbergs
tumor.
Enhanced aromatization of precursor androgens too
estrogens as in obesity, liver disease and hyperthy-
roidism.
7. What are the risks of estrogen replacement in meno-
pause?
1. Endometrial cancer
2. Breast cancer (only in few subsets)
3. Hypertension
4. Thromboembolic disease (raised factor VII, IX, X
decreased antithrombin III)
5. Increased incidence of gallstone formation due to
reduced chenodeoxycholate and high cholate levels
in the bile with raised excretion of cholesterol).
6. Mastodynia and breast engorgement.
7. Abdominal bloating
8. Headache (menstrual migraine).
8. Contraindications for estrogen replacement
Undiagnosed vaginal bleeding
Acute liver disease, chronic hepatic decompensa-
tion
Vascular-venous thrombosis
Seizure disorder
Estrogen receptor positive breast tumor
Hypertension, hyperlipidemia, migraine
Uterine fibroid, fibrocystic disease of breast.
9. Common causes of amenorrhoea
Pregnancy
Lactation Physiological
Menopause
Turners syndrome
114 Short Notes and Short Cases in Gynaecology

Imperforate hymen
Vaginal septum
Mullerian dysgenesis
Ashermans syndrome (intrauterine synechiae)
Hypothalamic defect (Kallmanns syndrome)
Pituitary dysfunction (atrophy, hyperprolactinemia)
Ovarian failure
Ovarian dysfunction (Stein-Leventhal syndrome)
Obesity, athletes.
10. Causes of primary ovarian failure.
Steroidogenic enzyme defects
True hermaphroditism
Gonadal dysgenesis
Ovarian resistance syndrome
Autoimmune oophritis
Postmumps, irradiation, chemotherapy
Idiopathic
11. The most dependable test to diagnose primary ovarian
failure.
Serum FSH. A value greater than 40 m IU/ml indicates
primary ovarian failure. A level below 40 m IU/ml indi-
cates hypothalamopituitary dysfunction necessitating
CT-MRI scan and prolactin estimation.
12. What is progestin challenge and its interpretation?
Progestin challenge indicates if ovary is producing
estrogen or not. When endometrium is ripe because of
ovarian estrogen, medroxy progesterone acetate 10 mg
daily for days is followed by vaginal bleeding. If not,
the patient has either no estrogen secretion from ovary
or is having Asherman syndrome. The latter only occurs
in patients having undergone D and C. A positive
progestin challenge test means patients is having
eugonadotropic eugonadism and needs exclusion of
thyroid, adrenal dysfunction, ovarian tumor and poly-
cystic ovary.
13. Management of patients, who bleed on progestin chal-
lenge.
Clomiphene citrate 0 mg daily for days; ovulation
occurs 10 days after last dose and bleeding follows
Case 2 115

2 weeks after. If no bleeding occurs clomiphene dose


may be increased upto 20 mg daily in 0 mg incre-
ments.
hMG + clomiphene when previous two methods fail.
Patients on hMG should have serial ultrasound and
estradiol estimation to know if too many follicles are
maturing, often causing marked ovarian enlargement
and ascites.
14. Treatment of progestin challenge negative patients.
hMg + clomiphene
LHRH
15. Is Wedge resection essential in polycystic ovary?
No, wedge resection is often followed by pelvic adhe-
sion of ovary and mechanical infertility. Patient is to be
tried first with hMG and clomiphene.
16. Which rare enzyme defects cause amenorrhoea?
17 alpha hydroxylase deficiency
17 to 20 desmolase deficiency.
17. What is premature ovarian failure?
Ovarian failure and thus amenorrhoea occurring before
age of 40 years. Such patients may have an immune
disorders (Hashimotos syndrome, Addisons disease,
hypoparathyroidism) or have 46 XX/46 YY karyotype.
Ovarian failure may result from irradiation and chemo-
therapy.
18. Drugs causing amenorrhoea.
a-methyldopa Stimulate prolactin secretion
Phenothiazines
Tricyclic antidepressants
Reserpine
Benzodiazepine
Opiates, barbiturates, estrogen
19. Which hormones are to be assayed first in a patient of
amenorrhoea?
TSH, T3, T4; because asymptomatic hypothyroidism
is often responsible for amenorrhoea. Menses start
with thyroid replacement.
116 Short Notes and Short Cases in Gynaecology

Prolactin level; as hyperprolactineamia can occur


without galactorrhoea.
20. How psychiatric disorders cause amenorrhoea?
Anorexia nervosa with rapid weight reduction causes
amenorrhoea. At less than 1 ideal body weight GnRH
release is hampered and at less than 2 ideal body weight
FSH and LH release is also abnormal. These patients
have low T3 levels because of impaired peripheral con-
version of T4 to T3.
21. Synonymous for polycystic ovarian disease (PCO).
Stein-Leventhals syndrome
Chronic hyperandrogenism
Chronic oligoovulation
Idiopathic hirsutism (these patients have normal
menses)
22. Criteria for diagnosis of PCO.
Premenarcheal onset of menstrual irregularity
Increased body weight
Chronic anovulation
Inappropriate gonadotropin secretion
Euprolactinemia
US showing ovaries enlarged and with multiple cysts.
23. Most common clinical manifestations of PCO.
Infertility
Hirsutism
Endometrial hyperplasia
24. What is pathological-pharmacological basis of PCO?
There is elevated LH with LH:FSSH of 3:1 (normal 1.5:1)
and causes hyperplasia of theca cells with elevated tes-
tosterone. Elevated androgens account for the elevated
total estrogen levels through peripheral conversion of
androstenedione to estrone. The cysts in PCO are atretic
follicles. In 50 percent cases, the androgen excess
involves both adrenal and ovarian glands. Spironolac-
tone can reverse hirsutism of PCO.
Case 2 117

25. When to do karyotyping?


Patients of primary amenorrhoea because 35 percent of
them are due to Turners syndrome and another 20 per-
cent due to mullerian agenesis.
26. Work-up for primary amenorrhoea
Pelvic ultrasound is to be done to look for presence of
uterus. If uterus is absent Karyotyping is advised. When
uterus is present and vagina is patent, breasts are exam-
ined. Absence of breast enlargement means work-up
for progestin negative secondary amenorrhoea (estro-
gen lack) whereas normal breast development indicates
work-up for secondary amenorrhoea.
Case 3

A 25-year-old lady is complaining of purulent vaginal


discharge, fever and pelvic pain for past days. P.V. examina-
tion reveals a normal size uterus with restricted mobility and
tenderness in fornices.
1. What is the most likely diagnosis?
Pelvic inflammatory disease.
2. What is pelvic inflammatory disease (PID)?
PID is synonymous with salpingitis, but can represent
any infection process involving upper genital tract like
endometritis, oophoritis, parametritis, and combination
of such entities.
3. Which clinical disorders mimic PID?
Acute appendicitis Mesenteric adenitis
Endometriosis Renal colic
Torsion of ovary Ischaemic colitis
Corpus luteum bleeding Tubal pregnancy
Pelvic adhesions
4. How to differentiate tubal pregnancy from PID?
1. Beta hCG estimationwhich is increased in tubal
pregnancy.
2. Pelvic ultrasound
5. Is cervical motion tenderness specific for PID?
Cervical motion tenderness is a feature of pelvic perito-
nitis. Hence, it may be elicited in tubal rupture, intraperi-
toneal bleed, appendicitis, etc.
6. How culdocentesis is useful in management of PID?
Obtaining peritoneal fluid samples by culdocentesis can
be useful in confirming diagnosis of PID. Microorgan-
isms can be isolated in three fourth of the patients. Gram
stain and culture sensitivity guide in selecting the anti-
biotic.
Case 3 119

7. Role of laparoscopy in diagnosis of PID.


Laparoscopy is indicated in any acute pelvic pain of
obscure etiology. The finding of swollen erythematous
tubes with purulent exudate in absence of another cause
for peritonitis is diagnostic of acute PID. the appendix
can be visualised in more than 88 percent cases. Endo-
metriosis cannot be missed.
8. Role of sperms in pathogenesis of PID.
Bacteria attach to spermatozoa and are thus able to
penetrate cervical mucus barrier. Hence women using
diaphragm or spermicides as contraception have a lower
incidence of PID.
9. Role of oral contraceptives in PID.
Women using oral contraceptives have lower incidence
of PID because cervical mucus is less voluminous, but
more tenacious, thus forming a formidable barrier for
bacteria and sperms.
10. Which women are at risk for acquiring PID?
Multiple sexual partners
IUD contraception
Gonococcal and chlamydial cervicitis
Sexual partner with gonococcal and chlamydial in-
fection
11. Routes of infection for PID.
1. Ascending infection from cervix and vagina.
2. Hematogenous spread e.g. tuberculous salpingitis.
3. Local spread from appendicitis.
12. Bacteria isolated from PID patients.
Gonococci Annaerobes
Chlamydia Bacteroid sp.
E. coli Peptococci sp.
Gardenella vaginalis Peptostreptococci sp.
H. influenzae
Proteus mirabilis
Klebsiella
13. Are women more vulnerable to PID during menstrua-
tion?
Yes. Loss of protective endocervical mucous plug and
availability of blood in genital passage, a good medium
120 Short Notes and Short Cases in Gynaecology

for growth of microorganism, enhance chances of PID


during menstruation.
14. When to advise ultrasound in a patient of PID?
Ultrasound is advised, when
1. Pelvic pain precludes through pelvic examination.
2. Tubo ovarian mass is suspected.
15. When to hospitalize a patient of PID?
Nulliparity with first attack of PID
Presence of pelvic mass
IUD with PID
Pregnancy
Uncertain diagnosis
Noncompliance or no response to drugs within 2
days.
16. Why nulligravida with acute PID are aggressively
treated?
Aggressive treatment with I V antibiotic in nulligravida
even with mild PID is justified because of potentially
serious sequelae in them like tubal scarring and infertil-
ity, ectopic pregnancy, and chronic pelvic pain.
17. Sequelae of PID.
Infertility (tubal block)
Tubal ectopic pregnancy
Chronic dyspareunia
Chronic lower abdominal pain
Ovarian dysfunction due to oophoritis/adhesion
The risk of infertility after one attack of PID is as high as
30 percent. Half of women with tubal ectopic pregnancy have
a history of PID.
Outpatient treatment of PID
Amoxycillin 3 gm with probenecid 1 gm
Or
Cefoxitin 2 gm or IM with probenecid 1 gm
Or
Ceftriaxone 250 mg IM
Followed by
Doxycycline 100 mg PO bid for 10 to 14 days
Or
Tetracycline 500 mg PO qid for 10 to 14 days
Or
Erythromycin 500 mg PO qid for 10 to 14 days
Case 3 121

18. When to treat a patient of PID as outpatient?


Temp. < 39C
Patient not toxic
No pelvic abscess/tubo-ovarian mass.
19. In hospital treatment of PID.
Metronidazole 1 gm IV bid with doxycycline 100 mg IV
bid unit afebrile for 2 days followed by metronidazole
and doxycycline in above doses PO to complete 14 days.
Doxycycline 100 mg IV bid with cefoxitin 2 gm IV qid
until afebrile for 48 hours or upto 4 days followed by
doxycycline 100 mg. PO bid for 10 to 14 days.
20. Definition of chronic pelvic pain in women.
Nonspecific pelvic pain of greater than 6 month dura-
tion not fully relieved by narcotic analgesics.
21. Common causes of chronic pelvic pain.
Endometriosis
Pelvic congestion
Postoperative adhesion
Pelvic serositis
Tubo-ovarian disease.
22. Diagnosis of tubo-ovarian abscesses.
Pelvic pain, fever
Adnexal mass in PV examination
Adynamic ileus
Ultrasound
History of IUD use, salpingitis.
23. Differential evaluation of fluid obtained by culdocente-
sis.
1. Blood
Ruptured ectopic pregnancy
Ruptured corpus luteum cyst
Retrograde menstruation
Acute salpingitis.
2. Pus
Ruptured tubo-ovarian abscess
Ruptured appendix
Ruptured diverticular abscess
122 Short Notes and Short Cases in Gynaecology

3. Cloudy
Pelvic peritonitis of any cause
Twisted ovarian cyst.
24. How to evaluate chronic pelvic pain?
Pelvic ultrasound
Through P.V. examination to identify site of pain and
trigger points
Laparoscopy
25. What is pelvic vascular congestion syndrome?
It is a dynamic vascular process similar to migraine and
is related to emotional disturbances.
Case 4

A young lady has been complaining of vulvar itching, dysuria


and whitish vaginal discharge. She is taking oral hypoglyce-
mic drugs.
1. What is your presumptive diagnosis?
Candidial vulvovaginitis in a diabetic.
2. Characteristics of normal physiologic vaginal dis-
charge.
A physiologic vaginal discharge consists of cervical
and vaginal secretions, epithelial cells and bacterial flora.
Normal vaginal pH is 3.8 to 4.2. A physiological white
discharge is usually white and odorless, does not cause
itching, burning or perineal discomfort.
3. What comprises normal vaginal flora?
Doderleins bacilli, an aerobic gram positive rod is the
most common bacteria. Others include streptococci,
staphylococci, diphtheroids, Gardenella vaginalis, E. coli,
anaerobes, mycoplasma and candida. However, tissue
invasion by these agents in a healthy woman is rare.
4. Common agents of vaginitis.
Candida (yeast and monilia)
Trichomonas vaginalis
Synergistic bacteria (bacterial vaginosis)
5. Characteristics of candidial vaginitis.
Vaginal and vulvar pruritus
White cottage-cheese discharge
KOH preparation shows yeast and hyphae
Culture in Nickersons medium in diagnostic
6. Treatment of candidial vaginitis.
Cream or suppository preparation of nystatin, clotri-
mazole, miconazole, terconazole, butoconazole for
3 to 7 days.
124 Short Notes and Short Cases in Gynaecology

Topical gentian violet or boric acid suppository


Treatment of diabetes when associated.
7. Characteristics of trichomonas vaginitis.
Malodorous discharge (fishy smell)
Greenish-gray frothy
Saline smear shows plenty fo WBC and the actively
motile flagellated trichomonas
8. Treatment of trichomonas vaginitis.
Metronidazole 2 gm. PO single dose to both partners
or tinidazole 2 gm.
Alternatively metronidazole 200 mg tid. PO for 7 days
to both partners.
Clotrimazole suppository for 7 to 14 days in addition
to metronidazole.
9. Characteristics of bacterial vaginosis.
Gray colored smelling discharge
Saline preparation shows clue cells, i.e. vaginal
epithelial cells studded with cocco bacilli (Gardenella
organisms gram-negative)
Positive Whiff test, i.e. application 10 percent KOH
to discharge emits fishy odor if Gardenella vaginalis
is the infecting agent
10. Treatment of bacterial vaginosis.
Metronidazole 500 mg. PO bid for 7 days or Ampicillin
500 mg PO qid for 7 days.
11. Decreases caused by Gardenella.
Vaginitis
Cervicitis
Postpartum endometritis
Septicemia after abdominal hysterectomy with gas-
eous crepitation in abdominal incision
12. Characteristic features of chlamydia infection.
Chlamydia trachomatis produces muco-purulent cervi-
citis with vaginal discharge but not vaginitis. As chlamy-
dia is intracellular, diagnosis is based on tissue culture
Case 4 125

or tests for chlamydial antigen. The infection should be


suspected in any patient of vaginal discharge with
numerous WBCs, but no clue cells yeast or trichomonas.
13. Choice of therapy in chlamydial infection.
Doxycycline 100 mg bid PO for 7 to 10 days or erythro-
mycin 500 mg. PO qid for 7 days.
14. Characteristic features of atrophic vaginitis.
Here the inflammation is secondary to vaginal mucosal
atrophy secondary to estrogen lack as in post meno-
pausal women. Itching, dryness, dyspareunia and often
vaginal bleeding are chief complains. Treatment is with
estrogen replacement or vaginal estrogen creams.
15. How common is genital herpes simplex?
Genital herpes is caused by HSV 2 virus. About one-
third of sexually active couples have G2 antibody, but
only 5 percent have past history of herpes, since infec-
tion goes unnoticed. The primary clinical attack of
herpes occurs with painful blisters and systemic symp-
toms but the organism innervates root ganglia (S3, S4),
where it lies dormant to produce recurrent attacks
throughout life.
16. What is difference between primary and recurrent
herpetic attacks?
Primary infection is widespread, takes longer time to
heal (12-21 days) and is associated with systemic signs
and symptoms (fever, headache, aseptic meningitis).
Recurrent attacks are of shorter duration (2-4 days) and
localized. HSV1 though primarily causes fever, blisters
on lips, it can also infect genital passage with mild attack
and its recurrences rapidly diminish in frequency.
17. Description or herpetic skin lesion.
Clusters of clear or yellow fluid filled vesicles with
erythematous base like dew drops on rose petal. In
genital area rather painful ulcerations, papules and even
folliculitis like lesions are more common. Herpetic cervi-
citis causes white-gray area of necrosis, often mistaken
for cancer. All these lesions are infectious to sexual
126 Short Notes and Short Cases in Gynaecology

contact. Infection occurs in half of close sexual con-


tacts as HSV does not appear to infect intact epithelium.
18. Can genital herpes spread without contact with her-
petic lesion?
Yes. Twenty-five percent of new infections are mediated
by contact with asymptomatic individual shredding
HSV from genital tract.
19. Common viruses incriminated in cancer of cervix,
vagina, vulva and penis.
HSV2
Human papilloma virus
20. Is it necessary to treat all primary attacks of genital
herpes?
Yes. Acyclovir 200 mg 5 times daily for 10 days helps in
viral clearance. Recurrent attacks being milder and short
lived, do not warrant acyclovir therapy, but to suppress
frequent recurrences (> 6 per year) acyclovir 400 mg
twice daily may be taken.
21. Which patients should receive IV acyclovir?
Herpes encephalitis
Herpes hepatitis
Extensive skin lesions with meningismus
22. How acyclovir works?
Within infected cell, acyclovir is activated by HSV
determined thymidine kinase. Activated acyclovir sup-
presses viral replication, but several HSV strains are
resistant to acyclovir. Development of resistance may
be prevented by suing full doses of acyclovir.
23. What are rather uncommon forms of vaginitis?
1. Desquamative inflammatory vaginitis.
2. Vaginitis emphysematosa by G vaginalis.
3. Pinworm and E. histolytica vaginitis.
4. Ureaplasma urealyticum.
5. Condylomatous vaginitis.
24. Role of human papilloma virus in genital tract disease.
Human papilloma virus causes condyloma acuminatun
of cervix, vagina, vulva, perineum and perianal areas. It
Case 4 127

can be exophytic or flat, spiked and inverted as in vagi-


nal epithelium and cervix.
25. Diagnostic clues to condylomatous vaginitis.
Vaginal discharge and pruritus
Koilocytes, i.e. cell with perinuclear faintly staining
cavity.
Culposcopic examination shows the lesions often
mistaken for intraepithelial neoplasia
The chief histological difference between dysplasia and
condyloma is the direction of progression of cellular atypia.
In dysplasia, the dysplastic cells move towards surface, but
in condyloma, they move towards basal membrane.
Case 5

A 35-year-old infertile, nulliparous lady has been complain-


ing of lassitude, palpitation and dyspnoea on exertion. Exami-
nation reveals pale conjunctiva and on questioning she gives
history of heavy periods for past two years.
1. What do you think of this case?
Menorrhagia with anaemia; Endometrial malignancy
needs exclusion.
2. Causes of menorrhagia.
Submucous myoma
Adenomyosis
Endometrial hyperplasia
Malignant tumors
Dysfunctional uterine bleeding
Complications of pregnancy
3. What is metrorrhagia?
Metrorrhagia is intermenstrual bleeding occurring in
endometrial polyps, and endometrial and cervical carci-
nomas. Ovulatory bleeding also occurs at midcycle.
4. What is metromenorrhagia?
It is irregular acyclical heavy bleeding, common to
cervical cancer.
5. Diagnostic work-up in metromenorrhagia.
Cervical, endocervical and vaginal cell cytological
Endometrial biopsy
Hysteroscopy
Pelvic ultrasound
6. Which type of patients are vulnerable to endometrial
cancer?
Obese
Nulliparous
Case 5 129

Infertile
Late menopause
Polycystic ovary disease
Ovarian granulose cell tumors
Estrogen intake
Diabetes and hypertension.
7. Stages of endometrial cancer.
1. Endometrial hyperplasia; cystic and adenomatous.
2. Carcinoma in situ.
3. Invasive carcinoma.
Twenty to twenty-five percent of patients with adeno-
matous hyperplasia and 50 percent cases with atypical
adenomatous hyperplasia will develop endometrial cancer, if
left untreated.
8. Staging of endometrial cancer.
IA Tumor limited to endometrium
IB Invasion of less than 1/2 myometrium
IC Invasion of more than 1/2 myometrium
IIA Endocervical glandular involvement only
IIB Cervical stromal invasion
IIIA Tumor invades serosa
IIIB Vaginal metastasis
IIIC Metastasis to pelvic, and/or para-aortic lymph
nodes
IVA Tumor invasion of bladder bowel mucosa
IVB Distant metastasis to inguinal and intra-abdomi-
nal lymph nodes
9. Prognostic factors in endometrial cancer.
Histologic differentiation and type
Stage of invasion and metastasis
10. Histologic types of endometrial cancer.
Adenocarcinoma
Adenoacanthoma
Adenosquamous carcinoma
11. Treatment of endometrial cancer.
Hysterectomy with bilateral salpingo-oophorectomy for
stage I disease. Peritoneal washing, cytology, pelvic
and para-aortic node sampling is done to decide about
adjuvant radiotherapy.
130 Short Notes and Short Cases in Gynaecology

12. Indications for adjuvant radiotherapy.


Lower uterine segment or cervical involvement
Poor histologic differentiation (solid/anaplastic)
Papillary, serous or clear cell histology
Stage IC.
13. Treatment of Stage II and advanced cases of endome-
trial cancer.
Intracavitary radium/cesium in combination with whole
pelvic radiation pre-operatively followed by hysterec-
tomy and salpingo-oophorectomy.
14. Role of progesterone in management of endometrial
malignancy.
1. Endometrial hyperplasia is reversible with timely
progesterone supplement.
2. Patients with advanced or recurrent endometrial
cancer with positive estrogen or progesterone recep-
tors can be treated with high dose progestin therapy,
in addition to surgery and radiation.
15. What is uterine papillary serous adenocarcinoma?
It is a variant of endometrial cancer but with histology
resembling ovarian serous carcinoma. It tends to spread
intraperitoneally, similar to ovarian cancer and has a
higher incidence of nodal and distant metastasis.
16. Clinical presentation of endometrial sarcoma.
Bleeding : Intermenstrual, hypermenorrhoea, post-
menopausal, pre-adolescent.
Mass : Rapid enlargement of uterus, or of a
leiomyoma.
Pain : Discomfort in the pelvis from pressure
on surrounding organs.
Age : At any age but commonly around 40
years.
Metastasis : Lungs.
17. Major types of sarcomas.
Leiomyosarcoma
Endometrial sarcoma
Case 5 131

18. Treatment of uterine sarcoma.


1. Control of haemorrhage with 400-500 rads to pelvis.
2. Radical hysterectomy with sapingo-oophorectomy
for localized low grade sarcoma.
3. Simple hysterectomy with salpingo-oophorectomy
with pre- or postoperative radiation.
19. Prognosis of Endometrial cancer.
Stage 5-year survival
Stage IA 75-90 percent
Stage IC 30-40 percent
Stage II 50 percent
Stage III 30 percent
Stage IV >5 percent
20. Treatment of endometrial hyperplasia.
Progesterone
Endometrial laser ablation under hysteroscopy.
Case 6

A 30-year-old healthy black woman complaining of dysmen-


orrhoea, on pelvic examination, reveals to have irregular
enlargement of uterus.
1. What is the presumptive diagnosis?
Uterine leiomyoma.
2. Which terminology is correctfibroid or leiomyoma?
The term fibroid is a misnomer since these tumors are
composed primarily of smooth muscle cells rather than
fibrous issue. Leiomyoma is the correct name for these
tumors.
3. General considerations about leiomyoma.
1. They are present in upto 20 percent of women in
their reproductive age.
2. They are the most common pelvic neoplasm in
females.
3. More common in black than white women.
4. They can produce wide spectrum of problems rang-
ing from infertility to haemorrhage.
5. Malignant transformation only occurs in 0.5 percent
cases.
6. Though mostly asymptomatic, they mask other
potentially lethal pelvic tumors.
4. Classification of leiomyoma.
Intramural : Tumor located in the uterine wall
Subserosal : Tumor just located beneath uterine
serosa
Submucous : Tumor growing just below the endo-
metrium (5-10 percent of myoma)
5. What is parasitic leiomyoma?
A pedunculated leiomyoma that becomes attached to
adjacent viscera or omentum and develops a secondary
blood supply.
Case 6 133

6. Are leiomyomas estrogen dependent?


Yes, leiomyomas usually occur in women during child
bearing age. They are absent before puberty and regress
after menopause. They respond to GnRH analog buse-
riline and nafariline. Estrogen receptors are higher in
myomas than in surrounding myometrium.
7. Symptoms commonly associated with leiomyomas.
Bleeding: Hypermenorrhoea, metror-
rhagia, dysmenorrhoea
Pain: From torsion or degeneration
Pressure symptoms: Ureteral obstruction, urinary
frequency, constipation
Infertility: Submucous variety interfer-
ing implantation
Hypertension and polycythemia have been associated
with leiomyoma.
8. How leiomyomas impair fertility?
A leiomyoma can impair, fertility be occlusion of endo-
cervical canal, by distrosion of fallopian tubes or by
changing endometrium to impair implantation. In addi-
tion, leiomyomas may complicate pregnancy be causing
abortions, premature labor or dystocia.
9. Types of degeneration in leiomyoma.
Atrophic
Hyaline
Cystic
Calcific
Septic
Carneous (red)
Myomatous (fatty)
Malignant transformation.
10. Special investigation in diagnosis and treatment of
leiomyoma.
Hysteroscopy for diagnosis of submucous leio-
myoma
Ultrasonography
MRI imaging, when malignancy is suspected
I V urography to know ureteral deviation/compres-
sion prior to pelvic surgery.
134 Short Notes and Short Cases in Gynaecology

11. Differential diagnosis of leiomyoma.


Ovarian tumor
Endolymphatic stromal meiosis
Tubo-ovarian mass
Endometriosis
Pregnant uterus.
12. How leiomyomas cause bleeding?
Some submucous myomas cause bleeding due to ulcer-
ation of endometrium over the area of myoma. More
likely, myomas cause change in venous drainage pattern
and vascular alteration of the endometrial surface. The
bleeding is certainly not due to increased endometrial
surface area.
13. Which factors influence treatment choice in myoma?
Patients age, parity, pregnancy status, desire for further
pregnancy, general health and symptoms, and the size,
location influence choice of treatment.
14. Treatment modalities for myoma.
Myomectomy when pregnancy desired
Hysterectomy
Excision of pedunculated myoma
GnRH analog to shrink large bleeding myomas
Radiotherapy to myomatous uterus, bleeding pro-
fusely in poor surgical risk patient
15. Gold standard for surgical treatment of myoma.
Uterus size more than 12 weeks or cervical myoma larger
than 3 to 4 cm.
16. Which myomas pose problem in surgical removal?
Large tumors pose difficult surgical problem of expo-
sure and vascular control, particularly if the mass is
cervical, intraligamentary or has displaced the ureter.
17. What is adenomyosis?
Presence of endometrial glands and stroma within
myometrium is called adenomyosis.
18. When to suspect adenomyosis?
Hypermenorrhea
Dysmenorrhea
Case 6 135

Diffuse globular firm enlargement of uterus


Nearly 30 percent cases are asymptomatic.
19. What is endolymphatic stromal myosis?
When the stromal component of endometrium, without
glands, invades the myometrium, the resulting tumor is
termed as endolymphatic stromal myosis or stromatosis.
It is not hormone dependent, hence oral contraceptives
accentuate pain or bleeding. Hysterectomy is curative.
20. When to suspect endometrial polyp?
Regularly recurring menorrhagia in a normal size
uterus
Minor pre-and postmenstrual bleeding
Bleeding continuing following D and C.
21. Diagnosis of endometrial polyp.
Hysteroscopy
D and C
Hysterosalpingography.
Case 7

A 25-year-old infertile female is complaining of dysmenor-


rhoea. PV examination reveals fixed retroverted uterus, with
asymmetric adnexal enlargement and tenderness.
1. What do you think of the case?
Endometriosis is a strong suspicion.
2. What is endometriosis?
Endometriosis is the growth of endometrial gland and
stroma outside the uterus. This ectopic endometrial tis-
sue is capable of infiltration, proliferation and spread to
remote sites.
3. Common sites of endometriosis.
Ovaries bilateral in 65 percent cases
Pelvic peritoneum
Anterior and posterior cul-de-sac
Uterosacral, round and broad ligaments
Fallopian tubes
Cervix, vagina and bowel in 10 percent cases
4. Infrequent sites of endometriosis.
Lungus, nasal mucosa, umbilicus, bladder, kidney, legs
and episiotomy sear.
5. Etiology of endometriosis.
1. Tubal regurgitation of menstrual epithelium.
2. Lymphatic or vascular metastasis.
3. Celomic metaplasia.
6. Clinical presentation of endometriosis.
Clinical manifestations depend upon site than extent
of disease
Cyclic pelvic pain, dysmenorrhoea, dyspareunia and
often infertility (30-40%) are the classical symptoms.
Case 7 137

7. Physical findings in patients of endometriosis


Diffuse pelvic or abdominal pain
Nodular thickening and tenderness along the utero-
sacral ligaments, in posterior cul-de-sac and posterior
surface of uterus
Variable degrees of induration and fixation of con-
tiguous pelvic structures
Fixed retroverted uterus
Scarring and narrowing of posterior vaginal fornix
Asymmetrical adnexal enlargement and tenderness.
8. Most dependable procedure for diagnosis.
Laparoscopy and biopsy.
9. Hormonal regimens used to suppress endometriosis.
1. Oral contraceptives to induce a state of pseudopreg-
nancy. The endometriotic tissue undergoes decidual
reaction, then necrosis and if finally absorbed.
2. Danazol, a synthetic derivative of testosterone, sup-
presses release of GnRH, inhibits multiple enzymes
in steroid synthesis pathway and interacts with other
steroid hormone receptor.
3. Medroxyprogesterone, a potent gonadotropin inhi-
bitor.
4. Buserelin and nofariline, GnRH analogs.
10. What is the present state of surgery in treatment of
endometriosis?
Conservative surgery, i.e. excision or laser ablation of
all accessible sites of active disease with preservation
of uterus, one fallopian tube and as much as of ovary is
undertaken. Laser is preferred because of minimal tis-
sue damage good healing without adhesion, bloodless
operating field and precision.
11. Difference between primary and secondary dysmenor-
rhoea.
Primary dysmenorrhoea is pelvic pain that occurs in
absence of pelvic disease. Secondary dysmenorrhoea
has always a cause.
138 Short Notes and Short Cases in Gynaecology

12. Types of primary and secondary dysmenorrhoeas.


Primary Secondary
Essential Acquired
Psychogenic Mechanical
Spasmodic Membranous

13. Typical history in primary dysmenorrhoea.


Pelvic cramping usually occurs after ovulation has be-
gun, usually 6 to 12 months after menarche. The pain
generally starts several hours before or with menstrual
flow and lasts for 48 to 72 hours. Associated symptoms
are headache, lower bachache, nausea and vomiting,
diarrhoea, dizziness, etc. The pain radiates to thighs.
14. Causes of secondary dysmenorrhoea.
Endometriosis
Adenomyosis
Pelvic inflammatory disease
Cervical stenosis
Fibroid, polyp
IUD use.
15. Mechanism of primary dysmenorrhoea.
Prostaglandin
Uterine ischaemia.
16. Management of primary dysmenorrhoea.
Antiprostaglandins
Induction of anovulation with oral contraceptives
Weight reduction if obese
Local hear, massage and regular exercise
Acupuncture and calcium channel antagonists
Transcutaneous electrical stimulation.
17. Investigations helpful in secondary dysmenorrhoea.
Laparoscopy
Ultrasound
Hysterosalpingogram
Dilatation and curregate
Hysteroscopy
Exclusion of STD by cervical culture.
Case 7 139

18. What facts in history should lead to search for second-


ary cause?
Heavy or frequent period
Recurrent pelvic infection
IUD use
Recent pelvic operation.
19. What other pelvic pathology can mimic dysmenorr-
hoea?
Appendicitis
Ruptured ectopic
Torsion of ovary
Haemorrhage into ovarian cyst or from the cyst
Miscarriage
Rupture/torsion of ovarian cyst
Red degeneration of myoma.
Case 8

A 40-year-old multiparous lady with multiple sexual partners


complains of vaginal bleed after intercourse. Vaginal exami-
nation reveals indurated irregular cervix.
1. What is your diagnosis?
Cervical malignancy.
2. Predisposing factors for cervical cancer.
Coitus at early age
Multiple sexual partners
HSV2 infection of cervix
Human papilloma virus 16, 18
Low socioeconomic status
Smoking
3. Types of cervical cancer.
Squamous carcinoma
Adenocarcinoma : 85 percent
Mixed adenosquamous carcinoma : 15 percent.
4. Pathogenesis of cervical cancer.
1. 90 percent of squamous cell carcinoma arise in intra-
epithelial layerat the squamo-columnar junction.
2. The per-invasive form of disease remains static for
7 to 10 years. 95 percent of such patients can be
cured by surgery of radiotherapy.
3. The tumor spreads to parametrium to involve ureters
and at that stage pelvic node metastasis is 30 to 40
percent.
4. Blood borne metastasis commonly occurs to liver.
5. Methods for early diagnosis of cervical cancer.
Pap smear
Colposcopically directed biopsy
Case 8 141

6. What is definition of microinvasion?


Microinvasion means invasion upto 3 mm or less
beyond basement membrane without pathologic evi-
dence of confluence or vascular space involvement
some authorities put this limit at 5 mm from base of
epithelium or 7 mm of horizontal spread.
7. Colposcopic findings of early cancer cervix.
1. White epithelium.
2. Mosaicism or coarse punctate pattern of the surface
capillaries, coma shaped, cork screw shaped and
spaghetti shaped capillary.
8. What is Schiller test?
Normal squamous epithelium of cervix contains glyco-
gen, which produces a brown colour in combination
with iodine. Non-staining indicates abnormal epithelium
scarring or cyst formation. This test is not specific for
cancer, but reveals non-glycogen containing epithelium.
Immature metaplastic epithelium is nonstaining.
9. Role of punch and cone biopsy in early cancer diagno-
sis.
The abnormal epithelial zone in culposecopic examina-
tion is subjected to punch biopsy. Cone biopsy is
indicated if lesion extends to cervical canal or in micro-
invasive carcinoma to know the depth of penetration.
10. Physical findings of cervical cancer in vaginal exami-
nation.
Infiltrative cancer produces enlargement, irregularity
and a firm cervix with firm parametrium
Often the growth is exophytic appearing as a cauli-
flower like friable bleeding lesion
Ulcerative growth of cervix mimicking chronic cervi-
citis.
11. Staging of cervical cancer.
Stage 0 Intraepithelial carcinoma
Stage IA Preclinical carcinoma, i.e. depth of infiltra-
tion less than 3 mm and horizontal spread
less than 7 mm
Stage IB Lesion confined to cervix
142 Short Notes and Short Cases in Gynaecology

Stage II Parametrium involved but pelvic walls free


involvement of vagina but not lower third
Stage III Extension to pelvic wall involvement of
lower third of vagina
Stage IV Involvement of bladder and rectum; spread
to distant organs.
12. 5-year survival in different stages of cancer cervix
Stage I 80-85 percent
Stage II 60-65 percent
Stage III 25-35 percent
Stage IV 8-14 percent
13. Pelvic and para-aortic node involvement in various
stages of cancer cervix.
Pelvic node Para-aortic node
Stage I 15-20 percent
Stage II 30-40 percent 10 percent
Stage III 45 percent

14. Microscopic types of epidermoid cervical cancer.


Large cell nonkeratinizing (best prognosis)
Large cell keratinizing
Small cell (Worst prognosis5-year survival 20%).
15. Features of advanced cervical cancer.
Weight loss, anaemia, cachexia
Pelvic pain radiating to thigh or hip
Involuntary passage of urine and faeces
Renal failure.
16. Best way of knowing extent of pelvic involvement
MRI
Lymphangiography
IVP
17. In a young patient with carcinoma in situ wanting to
preserve uterus, conization of cervix is enough pro-
vided.
1. There is ample margin of normal columnar epithelium
at the apex and squamous epithelium at the base of
the cone.
2. Patient is prepared for cytologic check-up every 3
months.
Case 8 143

3. Pap smear is negative for severe dysplastic and


malignant cells.
4. Cervical canal is not stenosed, in validating cyto-
logic sampling.
18. Treatment of microinvasive cervical cancer.
Extend hysterectomy, i.e. removal of uterus, ovaries,
parametrial tissue and upper vagina. Cure rate (5 years)
is around 95 percent.
19. Treatment of invasive cancer.
1. For stage I to IIA diseaseradiotherapy and sur-
gery are of equal value in terms of survival, but
immediate complications of radiotherapy are less
frequent and less severe.
2. For stages beyond IIA radiotherapy is the treatment
of choice.
3. Chemotherapy with cisplatin, doxorubicin and
bleomycin for recurrent disease, who cannot toler-
ate further surgery of radiotherapy.
20. Site of recurrence of cervical cancer after surgery
and radiotherapy,
After radical hysterectomy, one-third recurrence is in
pelvic side wall; and one-fourth recurrence is in central
pelvis. Recurrence after radiotherapy is usually in
parametrium.
21. Which patients are candidates for pelvic exenteration?
Pelvic exenteration is reserved for those cases of cervi-
cal cancer treated previously with radiotherapy, having
recurrence in central pelvis. The triad of sciatica, unilat-
eral leg edema and ureteral obstruction are indicative of
unresectable disease.
22. Role of adjuvant chemotherapy in patients of advanced
or recurrent disease.
Adjuvant chemotherapy is a radiosensitizer (e.g. 5
FU, hydroxycerea, cisplatin)
Prior adjuvant chemotherapy shrinks initial tumor
and takes care of micrometastasis.
144 Short Notes and Short Cases in Gynaecology

23. Planning dose of radiotherapy in cervical cancer.


1. 000 rads to pelvis in 25 sittings, 5 days a week in 5
weeks.
2. Intracavitary therapy, so as to deliver 6000 rads and
point A, i.e. a point-2 cm. Lateral to endocervical
canal and 2 cm above lateral vaginal fornix and 4000
rads to point B, i.e. an area 5 cm lateral to endocervi-
cal canal at the level of point A and represents lymph
bearing areas.
24. Role of acetic acid in culposcopic examination of
pelvis.
White epithelium is a result of an increased ratio of
nuclear material to cytoplasmic volume. Acetic acid
being dehydrating, accentuates this ratio. Hence, areas
with marginally elevated nuclear to cytoplasmic ratio
will now also be made prominent. Acetic acid is also
mucolytic and helps in cleansing of cervix.
25. What changes in cell constitutes metaplasia?
Increased nuclear to cytoplasmic ratio
Dense chromatin
Crowding of cells
Loss of polarity of cells in epithelial layer
Increased number of mitosis.
26. Koilocytes and their significance.
Koilocytes are found in epithelial layer and have pykno-
tic raisinoid multiple nuclei with perinuclear cytoplasmic
halo. They probably indicate human papilloma virus
infection.
27. Human papilloma virus and genital lesions.
Types 6 and 11 Causes raised condyloma
(wrat); mostly benign
Types 16 and 18 Causes flat warts, often
found in association
With invasive cervical
cancer
Types 30,31,33 and 35 Intermediate between
above two.
Case 8 145

28. Features of vulvar intraepithelial neoplasia (VIN).


Vulvar itching
Macular or papular, sharply demarcated white to red
or pigmented lesions
90 percent have positive HPV DNA and 20 to 30
percent give prior history of condyloma.
29. Clinical features of cervicitis.
Purulent vaginal discharge with disagreeable odor
Vulvar-vaginal irritation
Red edematous cervix
Tenderness on cervical motion
Positive laboratory studies for pathogens.
30. Causative agents of acute cervicitis.
Trichomonas vaginalis
Gonococci
Gardenella vaginalis
Herpes simplex
Chlamydia
31. Symptoms and sequelae of chronic cervicitis.
Leukorrhoea
Dyspareunia, dysmenorrhoea
Abortion, cervical dystocia
Invertility
Cervical malignancy (when HPV or HS are causative
agents)
Dysuria and frequency due to perivesical-lymphan-
gitis.
32. What is cervical erosion?
The occurrence of endocervical columnar mucosa on
portio vaginalis gives a red granular appearance, often
called cervical erosion.
33. Findings in chronic cervicitis.
Hypertrophied elongated cervix
Cervical laceration and eversion
Nabothian cysts.
34. Treatment of chronic cervicitis.
Electrocauterization
Cryosurgery
Laser surgery.
146 Short Notes and Short Cases in Gynaecology

35. Cystic abnormalities of cervix.


Nabothian cysts
Mesonephric cysts
Endometriosis.
36. What is nabothian cyst?
When a tunnel or cleft of tall columnar endocervical
epithelium is sealed off either through an inflammatory
process or as a result of epidermadization, the mucous
secretion is entrapped producing a cyst; clled naboth-
ian cyst.
37. Clinical features of cervical polyps.
Intermenstrual or post-coital bleeding
A soft red pedunculated protrusion from external OS
Microscopic examination confirms benign nature of
lesion.
38. What is botryoid sarcoma of cervix?
It is an embryonal tumor of cervix resembling small pink
or yellow grapes; contains striated muscles and other
mesenchymal elements. It is extremely malignant.
Case 9

A 50-year old lady, complaining of only heaviness in pelvis is


discovered to have an adnexal mass.
1. What do you think of this patient?
An ovarian neoplasm
2. How ovarian tumors cause pain?
Ovarian tumor causes pain due to pulsion, traction, dis-
tension (haemorrhage) or inflammation.
3. Gold standards about adnexal masses.
Any palpable ovarian mass in pre-menarcheal and
postmenopausal woman is abnormal.
All adnexal mass of 7 cm and above be immediately
removed
Posterior cul-de-sac masses are more commonly
malignant
A solid, bilateral, fixed mass of 10 cm and above is
often malignant
25 percent of genital cancers are ovarian but they
result in 50 percent of death
Germ cell tumors are more common in children and
young women whereas tumors of epithelial origin
are common in older women
20 percent of ovarin neoplasms are germ cell tumors
and 50 percent of them are malignant.
4. Common physiologic cysts of ovary.
Corupus luteum, follicular and theca lutein.
5. Why and how corpus luteum cysts are trouble shoot-
ers?
Corpus luteum cysts cause abnormal uterine bleeding
and often are confused with ectopic pregnancy, appen-
dicitis, diverticulitis, salpingo oophoritis and ovarian
torsion. Leaking corpus luteum causes leukocytosis and
low grade fever.
148 Short Notes and Short Cases in Gynaecology

6. How theca lutein cysts develop?


Theca lutein cysts form in response to excess endo-
genous gonadotropin levels. They are most commonly
associated with trophoblastic tumor (hydatidiform mole,
invasive mole, chorioade-noma detruens and chorio-
carcinoma).
7. Treatment of physiologic ovarian cysts.
Corpus luteum and follicular cysts generally regress or
disappear within 4 to 6 weeks.
8. Classification of ovarian germ cell tumors.
1. Teratomas
Immature
Mature
Monodermal (Struma ovarii)
2. Dysgerminoma
3. Endodermal sinus tumor
4. Embryonal carcinoma
5. Choriocarcinoma
6. Polyembryoma
7. Mixed forms.
9. Investigations in suspected germ cell tumors.
Ultrasound to know if solid, or cystic and the size
Karyotype
CT scanning for metastasis
Pelvic exploration
Study of tumor markers.
10. Tumor markers in ovarian germ cell tumors.
hCG AFP LDH CA125
Mixed germ cell tumor + + + +
Embryonal carcinoma + + + +
Dysgerminoma + + +
Immature teratoma + + +
Choriocarcinoma + + ?

11. International staging of ovarian cancer.


Stage I Growth limited to ovaries
Stage II Growth extending to pelvis structures
Case 9 149

Stage III Growth with peritoneal metastasis


Stage IV Distant metastasis.
12. Management of germ cell tumors.
Benign cystic teratoma (dermoids) should be
removed intact
Dysgerminomas should be removed and the uni-
nvolved ovary should be biopsied as the tumor is
bilateral in 30 percent cases. Infracolic omentectomy
and ipsilateral pelvic and para-aortic lymph node
resection is also done. Nodes can be irradiated as
dysgerminoma is highly radiosensitive. Disseminated
disease needs chemotherapy.
Non-dysgerminomatous malignant germ cell tumors
are confined to ovary and need salpingo-oophorec-
tomy, sub-total omentectomy, ipsilateral pelvic-para-
aortic node resection.
When associated with pregnancy (1-20 percent
cases), tumor reductive surgery be performed
Combination chemotherapy is a must to all patients
with germ cell tumors, with vinblastin, bleomycin and
cisplatin.
13. What are the epithelial (mesothelial) ovarian cancers?
Serous 35 percent, 40 to 60 percent are bilateral
Mucinous 10 to 20 percent
Endometroid
Clear cell
Transitional cell
Undifferentiated.
14. What are sexcord-stromal tumor?
1. Granulosastromal cell tumors
Granulosa cell tumor
Theca-fibroma
2. Sertoli-Leydig cell tumors (androblastoma).
3. Gynandroblastomas.
4. Unclassified.
15. What is Meigs syndrome?
The combination of clinically detectable ascites, hydro-
thorax, and a benign ovarian tumor, commonly an
150 Short Notes and Short Cases in Gynaecology

ovarian fibroma, is popularly known as Meigs syn-


drome.
16. What is Krukenbergs tumor?
This term should be restricted to ovarian neoplasmas
characterized by mucin containing signet ring cells, with
a sarcoma like stromal hyperplasia. The primary tumor
is most frequently from stomach and less commonly
from breast, colon and biliary tract.
17. Malignant tumors of oviduct.
Primary carcinoma
Mixed mesodermal tumor
The primary carcinoma is bilateral in half of the cases and
histologically can be papillary, papillary, papillary-alveolar
and alveolar. Treatment is with surgical removal along with
uterus and ovary followed by adjunctive chemotherapy.
Case 10

A 21-year old working lady with one living daughter aged


1 year has visited Gynaec outpatients for help, as she wants
to have her next baby after 4 years.
1. What is her need?
Contraception
2. Methods of contraception available to women.
Oral contraceptives
Intrauterine devices (IUD)
Physical barriers (diaphragm, spermicides, jelly, cer-
vical caps)
Subcutaneous implants of progesterone.
3. Failure rates of above contraception methods.
Method Failure rate per 100
Women years
Oral combination contraceptives 0.16-0.30
IUD 1.2-3
Diaphragm 2
Condom 3.54

4. Mechanism of action of oral contraceptives.


1. Prevention of ovulation by interference with GnRH
release and suppression of LH and FSSH.
2. Formation of thick scanty viscid cervical mucus that
impairs sperm penetration.
3. Altered motility of uterus and tubes, impairing sperm
and egg transport.
4. Decreased endometrial glycogen production.
5. Alteration of ovarian response to gonadotropins.
5. Contraindications for contraceptive use.
History of or presence of thrombophlebitis, throm-
boembolic disease, cerebrovascular disease
152 Short Notes and Short Cases in Gynaecology

Significantly impaired liver function


Known or suspected carcinoma of breast
Undiagnosed abnormal uterine bleeding
Known or suspected pregnancy
Estrogen dependent neoplasm.
6. Guidelines of oral contraceptive use in patients with
cardiovascular disease.
Oral contraceptives as such do not increase risk of
developing IHD, unless other risk factors like
1. Vascular disease
2. Hypertension
3. Diabetes mellitus
4. Hypercholesterelemia are also present. A significant
risk is there in women over 3 years of age, who smoke.
7. Effect of oral contraceptives on lipid metabolism.
The estrogen component of oral contraceptive agents
stimulates an increase in HDL cholesterol. The proges-
tin component decreases level of HDL cholesterol.
Hence, baseline lipid profile must be done in women
particularly those with family history of hyperlipidemia
before prescribing oral contraceptives.
8. Relationship between contraceptive use and mali-
gnancy.
There is no evidence that estrotens initiate the deve-
lopment of carcinoma from normal breast tissue or
increase the risk in those with family history of breast
cancer. Oral contraceptive users have a lower incidence
of non-malignant breast disease. There appears to be
no increased risk of endometrial cancer also. Incidence
of hepatocellular adenoma is increased in contraceptive
users, i.e. 1 per 30,000 to 50,000 women per year.
9. Time of return of ovulatory function after discontinua-
tion of oral contraceptives.
Approximately 45 percent of patients ovulate with the
first cycle after discontinuation, while 75 percent ovu-
late within the first 2 cycles of discontinuation.
Case 10 153

10. Influence of sex hormones on pregnancy outcome.


Progestins have been reported to increase incidence of
VACTERL syndrome. (Vertebral and cardiac, esophageal,
renal and limb anomalies).
Estrogen, most specifically diethylstilbestrol, has
been associated with clear cell adenocarcinoma of cer-
vix, cervical adenosis and cervical incompetence.
11. Methods to predict time of ovulation.
1. 14 days before the onset of next menstrual period.
2. Dip in basal body temperature followed by rise.
3. Upward surge in serum levels of luteinizing hormone.
12. Advantage of oral contraceptive use.
Reduction in benign breast disease, retention cyst
of ovaries, deficiency anaemia.
One-half less risk of developing endometrial cancer.
40 percent less risk of ovarian cancer.
Reduction in incidence of dysmenorrhoea
Protection against osteoporosis, ascending pelvic
infections and risk of toxic shock syndrome.
13. Advantage of daily mini progestin pill.
No conventional side effects of estrogen
Elimination of anovulatory bleeding abnormalities as
ovulation still takes place, but not conception
Convenient daily intake
0.35 mg of progestin per day provides good contracep-
tion, while maintaining reasonably normal ovulation.
14. Contraindications for use of intrauterine devices.
1. Pregnancy.
2. Severe cervicitis.
3. Malignancy of genital tract.
4. Significantly abnormal Pap smear.
5. Uterine bleeding of unknown cause.
6. Congenital or acquired distortion of uterine cavity.
7. Acute or subacute salpingitis.
8. Stenosed cervical canal.
9. History of ectopic pregnancy.
154 Short Notes and Short Cases in Gynaecology

15. Disadvantages and side effects of IUD.


1. Pregnancy rate of 1-3 percent depending upon nature
of IUD; lowest with double coil and progesterone T.
2. Expulsion rate of 7 to 20 percent, but lowest with
progesteron T.
3. Bleeding and pain.
4. Ectopic pregnancy
16. Indications for removal of an IUD.
1. Severe uterine cramps.
2. Excessive or persistent uterine bleed.
3. Perforation of uterus (laparoscopic or colpotomy).
4. Prolapse of device through cervical canal.
5. Salpingitis.
6. Intrauterine pregnancy occurring concurrently.
17. Mechanism of contraceptive action of IUD.
Exact mechanism is unknown. In most instances, it does
not prevent fertilization, but changes the endometrial
environment so that, the blastocyst cannot become
implanted. It probably interferes with enzyme action.
Copper is known to elicit leukocyte infiltration into myo-
metrium.
18. Indications for induced abortion.
Obstetrics
Prior uterine injury, severe prolapse
Cardiovascular
Aneurysm, systemic/pulmonary hypertension,
severe heart failure.
Others
Chronic renal disease, single kidney
Cholestatic jaundice of pregnancy
Grand mal epilepsy, multiple sclerosis, SLE, PAN,
rheumatoid arthritis
Downs syndrome or other serious congenital mal-
formation diagnosed antenatally
Maternal exposure to teratogens, ionizing radiation,
X-rays
Maternal schizophrenia, mental retardation
Victim of rape
Two or more living children
Case 10 155

Decision to carry or not to carry a pregnancy is the basic


right of every woman. Hence abortion is like elective cos-
metic surgery.
19. What is centchroman and its role as a contraceptive?
Centchroman is dimethyl phenyl methoxychroman
hydrochloride, a nonsteroidal oral contraceptive taken
twice a week for first 3 months and then once weekly.
Centchroman appears to manifest its contraceptive
action by accelerating embryo transport and develop-
ment and suppressing uterine preparation for implanta-
tion, therapy producing asynchrony between embryo
and uterine development, a critical requirement for
implantation.
20. Contraindications for use of Centchroman.
Absolute contraindications
Hepatic dysfunction
Polycystic ovary syndrome
Cervical hyperplasia
Relative contraindications
Severe allergy
Chronic ailments like tuberculosis, renal failure.

Vous aimerez peut-être aussi